Vous êtes sur la page 1sur 68

FORMATION MDICALE AU CAMEROUN

NATIONAL COMPREHENSIVE CLINICAL AND THERAPEUTIC EXAMINATION FOR


MEDICAL TRAINING IN CAMEROON
1 PARTIE
EPREUVES DE MEDECINE INTERNE - PEDIA TRIE ET SANTE
PUBLIQUE,

Dure : 3heures

1st PART
EXAMINATION OF INTERNAL MEDICINE - PEDIATRICS AND PUBLIC
HEALTH,
Duration: 3hours
12 Septembre 2014
CAHIER DE QUESTIONS
QUESTION BOOKLET
NOM
NAME
PRENOM
SURNAME
N* RECEPISSE
RECEIPT N
N DE TABLE SERIES
N
DATE

SIGNATURE

CE CAHIER COMPORTE 100 QUESTIONS. LES REPONSES DOIVENT ETRE


ECRITES UNIQUEMENT SUR LA FEUILLE-REPONSE. CE CAHIER DOIT
ETRE REMIS A LA FIN DE LEPREUVE AVEC LA FEUILLE-REPONSE.

THIS MCQ IS MADE OF 100 QUESTIONS. ALL YOUR ANSWERS MUST BE


WRITTEN ONLY ON THE ANSWER SHEET. THIS BOOKLET SHALL BE GIVEN
BACK WITH THE ANSWER SHEET.

Instructions: Questions 1-14 : For each clinical scenario below, suggest the most appropriate
clinical findings from the proposed options. / Pour chaque scenario clinique ci-dessous,
slectionnez la rponse la plus approprie parmi les options proposes.

1. A 63 year old man with a history of high blood pressure presents at emergency with sudden
onset tearing chest pain radiating to the back.
Pulmonary embolus
Pneumothorax
Aortic dissection
Pericarditis
Angina
Un homme de 63 ans aux antcdents dhypertension artrielle se prsente aux
urgences avec une douleur thoracique dchirante dinstallation soudaine sirradiant au
dos.
Embolie pulmonaire
Pneumothorax
Dissection aortique
Pricardite
Angine de poitrine
A.
B.
C.
D.
E.
1.

A.
B.
C.
D.
E.

2. A 45 year old man complains of cold numb fingers and difficulty in swallowing. On
examination he has tight skin, thickening of the fingers and telangiectasia.
A. Bechets disease
B. Systemic sclerosis
C. Reiters syndrome
D. Primary Raynauds disease
E. Sjogrens syndrome
2. Un homme de 45 ans se plaint des doigts insensibles et difficults avaler. A lexamen, il
prsente une peau rigide et tendue, un paississement des doigts et des tlangiectasies.
A. La maladie de Bechet
B. Sclrodermie
C. Syndrome de Peiter
D. La maladie primitive de Raynaud
E. Syndrome de Sjgren
3. Gastro-duodenal ulcer can cause the following complications except one. Which one?
A. Haemorrhage (Maelena and/or haematemesis)
B. Perforation into the peritoneum (Peritonitis)
C. Portal hypertension syndrome
D. Malignant transformation
E. Duodeno-pyloric
steosis
3. L'ulcre gastro-duodnal peut entraner les complications suivantes sauf une,
laquelle ?
A. Hmorragie (mlaena et/ou hmatmse).
B. Perforation en pritoine libre (pritonite)
C. Syndrome dhypertension portai e
D. Dgnrescence maligne.
E. Stnose duodno-pylorique.

4. In community acquired pneumonia, the following signs exist except one. Which one?
A. An infectious syndrome
B. A clinical syndrome of lung consolidation
C. An opacity in the lung parenchyma
D. Dullness of the side (flank)
E. Hyperleucocytosis
4. Dans la pneumopathie communautaire il existe toujours les signes suivants sauf un. Lequel ?
A.
B.
C.
D.
E.

Un syndrome infectieux
Un syndrome clinique de condensation pulmonaire
Une opacit parenchymateuve
Une matit des flancs
Une hyperleucocytose

5. Systemic corticotherapy is commonly used in dermatology. In which of the following diseases


is it contra-indicated?
A. Pemphigus
B. Pustular psoriasis
C. Acute dermatomyositis
D. Bullous pemphigoid
E. Acute lupus erythematosis
5. La corticothrapie gnrale est frquemment utilise en dermatologie. Dans quelle affection estelle contre-indique ?
A.
B.
C.
D.
E.

Pemphigus
Psoriasis pustuleux
Dermatomyosite aigu
Pemphigode huileuse
Lupus rythmateux aigu

6. Among the following metals which one is often responsible for contact eczema?
A. Iron
B. Gold
C. Silver
D. Copper
E. Nickel
6. Quel mtal est le plus souvent responsable d'un eczma de contact ?
A.
B.
C.
D.
E.

Le fer
L'or
L'argent
Le cuivre
Le nickel

7. Anemia is said to be regenerative when the reticulocyte level is


A. Superior to 25000/mm3
B. Superior to 75000/mm3
C. Superior to 120 000/mm3

D. Inferior to 25000/mm3
E. Between 25000/mm3 and 75000/mm3
7. Une anmie est dite rgnrative lorsque le taux de rticulocytes est :
A.
B.
C.
D.
E.

Suprieur 25 000/mm3
Suprieur 75 000/mm3
Suprieur 120 000/mm3
Infrieur 25 000/mm3
Compris entre 25 000/mm3 et 75 000/mm3

8. Among following etiologies of a transient ischaemic cerebral accident of a young subject


which one is the commonest?
A. Cerebral tumour
B. Thrombosis of a cerebral artery
C. Embolus of cardiac origin
D. Embolus of aortic origin
E. E. Embolus of carotid origin
8. Parmi les tiologies suivantes dun accident ischmique crbral transitoire du sujet jeune laquelle
est la plus frquente ?
A.
B.
C.
D.
E.

Tumeur crbrale
Thrombose dune artre crbrale
Embolie dorigine cardiaque
Embolie dorigine aortique
Embolie dorigine carotidienne

9. The diagnosis of Alzheimers disease has the following features except one. Which one?
A memory disorder
A disturbance of at least four other cognitive functions
An important disturbance of the daily activities of the patient
Insidious onset of symptoms, progressive and inexonerable worsening
The presence of risk factors: cranial trauma, age>65 years, female sex, low level of
education, past history of myocardial infarction.
9. Le diagnostic clinique de la Maladie dAlzheimer comprend les lments suivants sauf un.
Lequel ?
A.
B.
C.
D.
E.

A.
B.
C.
D.
E.

Un trouble de la mmoire,
Une perturbation dau moins quatre autres fonctions cognitives,
Une perturbation importante des activits quotidiennes du patient,
Dbut des symptmes insidieux, aggravation progressive et inexorable,
La prsence de facteurs de risque : traumatisme crnien, lge > 65 ans, le sexe fminin,
le faible niveau dducation, un ATCD dinfarctus du myocarde.

10. Concerning neuro-meningeal cryptococcosis, only one of the following statements is true.
Which one?
A.
B.
C.
D.

Is often observed in patients with a CD4 cell count >100/ JJ.1


Mortality is very high (20 to 37%) despite well conducted treatment
Concerns only patients infected by HIV no matter the clinical stage
Diagnosis is done by CT scan of the brain with injection of contrast material

E. Treatment comprises a bolus dose of Fluconazole (400mg/day) and a maintenance dose


(200mg/day)
10. A propos de la cryptococcose neuromninge une seule des propositions suivantes est vraie,
laquelle ?
A.
B.
C.
D.
E.

Est surtout rencontre chez des patients avec CD4> 100/fil,


La mortalit est trs leve (20 37%) malgr un traitement bien conduit,
Concerne uniquement les patients infects par le VIH, quelque soit le stade clinique
Le diagnostic se fait par le scanner crbral avec injection de produit de contraste,
Le traitement comprend une dose de charge de Fluconazole (400mg/jour) et une dose
dentretien (200 mg/jour).

11. The management of peripheral neuropathies due to anti-retroviral (ARV) consists of the
following measures except one. Which one?
A. Vitamin B complex
B. Stoppage of neurotoxic ARV (Stavudine, Didanosine)
C. Anti-epileptics (Carbamazepin, Pregabalin)
D. Physiotherapy to favour recuperation
E. Neurologic antalgics
11. La prise en charge des neuropathies priphriques lies aux ARV comprend les mesures
suivantes sauf une. Laquelle ?
A.
B.
C.
D.
E.

Larrt des ARV neurotoxiques (Stavudine, Didanosine),


Des antipileptiques (Carbamazpine, Prgabaline),
Les complexes vitaminiques B,
La physiothrapie pour favoriser la rcupration,
Des antalgiques neurologiques (Amitryptiline).

12. You are called to assist a patient who is having a generalized tonico-clonie crisis. What should you
do immediately?
A. Run to get Valium in the pharmacy
B. Put an object in his mouth to prevent him from swallowing his tongue
C. As soon as possible, lay him down on his side and wait for him to gain consciousness
D. Give him some water to drink
E. Transport him immediately to hospital
12. Vous tes appels au secours d'un patient qui fait une crise convulsive gnralise tonico- clonique.
Que faire dans limmdiat ?
A.
B.
C.
D.
E.

Courir pour chercher du Valium en pharmacie,


Mettre un objet dans sa bouche pour viter qu il avale sa langue,
,
Ds que possible, lallonger sur le ct et attendre son retour la conscience,
Lui donner un peu deau boire,
Le transporter immdiatement lhpital,

13. The following constitute elements of management of cerebral toxoplasmosis except one. Which
one?
A. Association of Sulfadiazine, Pyrimethamine and Lederfoline
B. Treatment duration is at least 6 months and can be prolonged in case of partial response
C. Putting patient on antiretrovirals (ARV)

D. Physiotherapy to favour recuperation in case of motor defect


E. Intravenous Corticosteroids (10 to 15 days) and oral anti-epiieptics in case of hypertension
13. Les lments de prise en charge thrapeutique de la toxoplasmose crbrale sont les suivants sauf un.
Lequel ?
A.
B.
C.
D.
E.

L association Sulfadiazine, Pyrimthamine et Lederfoline,


La dure du traitement est dau moins 6 semaines, prolonger en cas de rponse partielle,
La mise sous and rtroviraux (ARV),
La physiothrapie pour favoriser la rcupration en cas de dficit moteur,
Des corticodes IV (10 15 jours) et antipileptiques par voie orale en cas dhypertension
intracrnienne.

14. Diagnostic criteria for tuberculosis meningitis are the following except one. Which one?
A. A febrile meningeal syndrome
B. Notion of contact with a person with active tuberculosis
C. Presence in the CSF of high levels of proteins and cells
D. Normal levels of glucose and chlorides in the CSF
E. Presence of acid-fast bacilli (AFB) in Lowemstein culture media
14. Les critres diagnostiques de la mningite tuberculeuse sont les suivants sauf un. Lequel ?
A. Un syndrome mning fbrile,
B. La notion de contage tuberculeux,
C. La prsence dans le LCR dune lvation des protines et des cellules,
D. Un taux de glucose et de chlore normal dans le LCR
E. La prsence de BAAR dans la culture sur milieux de Lowenstein.
Instructions ; Questions 15-18For each clinical presentation bellow, give the most likely cause for
the clinical findings from the proposed options. / Pour chaque prsentation clinique ci- dessous,
slectionnez la rponse la plus approprie parmi les options proposes.
15. A 25 year old man has been referred to a rheumatology clinic with multiple painful stiff
joints and uveitis. He also complains of ulcers on his penis and mouth.
A. Bechets disease
B. Reiters syndrome
C. Sjogrens syndrome
D. Polymyalgia rheumatica
E. Relapsing polychondritis
15.Un homme de 25 ans a t rfr au service de rhumatologie avec des multiples articulations
raides et douloureuses ainsi quune uvite. Il se plaint aussi dulcres sur son pnis et dans sa
bouche.
A.
B.
C.
D.
E.

La maladie de Bechet
Le syndrome de Reiter
Le syndrome de Sjogrn
Polymyalgie rheumatica
Polychondrite rcidivante

16. A 45 year old woman presents with dryness in the eyes and mouth. Her Schirmers test is
positive.
A. Reiters syndrome
B. Primary Raynauds disease.
C. Sjogrens syndrome

D. Bechets disease
E. None of the above.
16.Une femme de 45 ans prsente une scheresse de ses yeux et de sa bouche. Son test de
Schirmer est positif ' '
,
A.
B.
C.
D.
E.

Le syndrome de Reiter
Maladie de Raynaud primaire
Le syndrome de Sjogrn
La maladie de Bechet
Polychondrite rcidivante

17. A 25 year old man presents with enlarged painless lymph nodes in the neck. His
peripheral blood film shows Reed-Sternberg cells.
A. Burkitts lymphoma
B. Hodgkins lymphoma
C. Myeloma
D. Acute lymphoblastic leukaemia
E. None of the above
17. Un homme de 25 ans prsente des adnopathies cervicales non douloureuses. L'examen du
frottis de son sang priphrique montre des cellules de Reed-Sternberg.
A.
B.
C.
D.
E.

Lymphome de Burkitt
Lymphome de Hodgkin
Mylome
Leucmie lymphoblastique aigu
Aucune des propositions ci-dessus

18. A 45 year old man presents with fever, weight loss, tiredness and gout. On examination
there is splenomegaly. White blood cell count is 112xl09/l. The Philadelphia chromosome is
detected.
A. Myeloma
B. Chronic lymphocytic leukaemia
C. Acute lymphoblastic leukaemia
D. Chronic myeloid leukaemia
E. None of the above
18. Un homme de 45 ans prsente une fivre, perte de poids, fatigue et la gote. A lexamen, il ya
une splnomgalie. Le taux de leucocytes est de 112 x l(?/l. Le chromosome Philadelphie est
dtect.
A.
B.
C.
D.
E.

Myomie
Leucmie lymphocytaire chronique
Leucmie lymphoblastique aigu '
Leucmie mylode chronique,
Aucune des propositions ci-dessus nest vraie

Instructions: Questions 19-23 :For each clinical scenario below, suggest the most appropriate
management from the proposed options. / Pour chaque scenario clinique ci-dessous, slectionnez
la prise en charge la plus approprie parmi les options proposes.

19. A 22 year old HIV positive individual on anti-retroviral therapy presents with
pneumocystis jeroveci pneumonia.
A. Intravenous benzylpenicillin
B. Intravenous ceftazidime
C. Intravenous flucloxacillin
D. Intravenous co-trimoxazole
E. Intravenous cefuroxime + erythromycin
19.Un individu de 22 ans, VIH positif sous traitement anti-rtroviral prsente une pneumonie
pneumocystis jeroveci.
A.
B.
C.
D.
E.

Benzyl pnicilline par voie intraveineuse


Ceftazidime par voie intraveineuse
Flucloxacillin par voie intraveineuse
Cotrimoxazole par voie intraveineuse
Cefuroxine et erythromycine par voie intraveineuse

20. A 19 year old man contracts pneumonia with symptoms of headache, fever and dry
cough. Serology shows evidence of Chlamydia infection.
A. Oral flucloxacillin
B. Oral erythromycin G. Oral ciprofloxacin
C. Oral amoxicillin
D. Oral rifampicin
20. Un homme de 19 ans contracte une pneumonie avec une symptomatologie de cphales,
fivre et une toux sche. La srologie met en vidence une infection chlamyienne.
A.
B.
C.
D.
E.

Flucloxacillin par voie orale


Erythromycine par voie orale
Ciprofloxacine par voie orale
Amoxicilline par voie orale
Rfampicine par voie orale

21. Standard therapy for community acquired pneumococcal pneumonia not requiring hospital
admission.
A. Oral flucloxacillin
B. Oral tetracycline
C. Oral ciprofloxacin
D. Oral amoxicillin
E. Cotrimoxazole
21.Lantibiothrapie standard pour la prise en charge d unepneumonie communautaire ne
ncessitant pas une hospitalisation.
A.
B.
C.
D.
E.

Flucoxacilline per os
Tetracycline per os
Ciprofloxacine per os
Amoxicilline per os
Cotrimoxazole per os

22. A previously healthy 65 year old smoker with early COPD complains of shortness of breath on
exertion.

A. Inhaled salbutamol
B. Oral aminophylline
C. Long term oxygen therapy
D. D .Intravenous salbutamol
E. Intravenous salmeterol
22. Une tabagique de 65 ans antrieurement sain avec une BPCO dbutante se plaint
dessoufflement leffort.
A.
B.
C.
D.
E.

Salbutamol inhal
Aminophylline per os
Oxygenothrapie au long cours
Salbutamol par voie intraveineuse
Salmeterol par voie intraveineuse

23. A 22 year old student with mild asthma that needs treatment for occasional early
morning wheeze.
A. Low-dose oral aminophylline
B. Oral prednisolone
C. Inhaled beclomethasone
D. Nebulized salbutamol
E. Inhaled salbutamol
23. Un tudiant de 22 ans avec asthme lger a besoin dun traitement pour une respiration
sifflante occasionnelle au petit matin.

A.
B.
C.
D.
E.

Aminophylline faible dose per os


Prednisolone per os
Bclomethase inhal
Salbutamol nbulis,
Salbutamol inhal

Instructions: Questions 24 : For each description of a chest radiograph below, give the most
likely cause for observed markings from the options proposed. / Pour chaque description de la
radiographie du thorax, choisissez la cause la plus probable parmi les options proposes.
24. Kerley B lines, bat-wing shadowing, prominent upper lobe vessels, cardiomegaly.
A. Right ventricular failure
B. Aortic stenosis
C. Mitral stenosis
D. Left ventricular failure
E. Post-fracture fat embolism
24. Les lignes de Kerley B, opacit en aile de papillon, revascularisation des sommets et
cardiomgalie
A.
B.
C.
D.
E.

Insuffisance ventriculaire droite


Rtrcissement aortique
Rtrcissement mitral
Insuffisance ventriculaire gauche
Embolie graisseuse post-fracture

Instructions: Questions 25-29 : For each clinical scenario below, give the most likely cause for
the clinical findings from the proposed options. / Pour chaque scenario clinique ci-dessous,
slectionnez la rponse la plus probable parmi les options proposes
25. A 65 year old smoker presents with shortness of breath, gallop rhythm and production of
pink frothy sputum.
A. Tuberculosis (TB)
B. Bronchiectasis
C. Goodpastures syndrome
D. Pulmonary oedema
E. Pulmonary embolus
25. Un tabagique de 65 ans prsente un essoufflement, un gallop et production dune
expectoration rose et mousseuse.
A.
B.
C.
D.
E.

Tuberculose
Dilatation de bronches
Syndrome de Good pasture
Oedeme pulmonaire
Embolie pulmonaire

26. A 22 year old man presents with fever, night sweats, weight loss and cough productive of
cupfuls of blood. Ziehl-Neelsen stain is positive for acid fast bacilli.
A. Tuberculosis
B. Polyarteritis nodosa
C. Goodpastures nodosa
D. Wegeners granulomatosis
E. Churg-Strauss syndrome
26. Un homme de 22 ans prsente une fivre, sueurs nocturnes, perte de poids et une toux
productive dune hemoptysie abondante. La coloration par Ziehl-Neelsen de crachats est positive
pour les bacilles acido-alcoolo-rsistants.
A.
B.
C.
D.
E.

Tuberculose
Polyarterite noueuse
Syndrome de Good pasture
Granulomatose de Wegener
Syndrome de Churg et Strauss

27. A 63 year old man presents to emergency with weight loss, cough, haemoptysis and shortness
of breath. On examination, he is anaemic, clubbed and apyrexial.
A. Bronchogenic carcinoma
B. Fibrosing alveolitis
C. Extrinsic allergic alveolitis
D. Cystic fibrosis
E. Pulmonary oedema
27.Un homme de 63 ans se prsente aux urgences avec perte de poids, totix, hmoptysie et
essoufflement. A lexamen il est anmique, prsente un hippocratisme digital et est apyrtique.

A. Carcinome bronchogenique
B. Alvolite fibrosante

C. Alvolite allergique extrinsque


D. Mucoviscidose
E. dme pulmonaire
28. Radio femoral delay of pulse in a patient with hypertension
A. Coarctation of aorta
B. Cardiac tamponade
C. Aortic stenosis
D. Atrial fibrillation
E. Atrial flutter
28. Un retard radio-femoral du pouls chez un patient hypertensif
A.
B.
C.
D.
E.

Coarctation de Vaorte
Tamponade cardiaque
Rtrcissement aortique
Fibrillation auriculaire
Flutter auriculaire

29. Pulsus paradoxus, jugular venous pressure rises on inspiration, heart sounds muffled.
A. Coarctation of the aorta
B. Cardiac tamponade
C. Gauchers disease
D. Mixed mitral valve disease
E. Mixed aortic valve disease
29. Pouls paradoxal, augmentation de la pression veineuse jugulaire linspiration, bruits du cur
assourdis
A.
B.
C.
D.
E.

Coactation de laorte
Tamponade cardiaque
Maladie de Gaucher
Maladie mitrale
Maladie aortique

Instructions: Questions 30-33 : For each clinical scenario below, suggest the most appropriate
management from the proposed options. /Pour chaque scenario clinique ci-dessous, slectionnez
la prise en charge la plus approprie parmi les options proposes.
30. A 65 year old man with heart failure requires rate control to treat coexisting atrial
fibrillation.
A. Lidnocaine (lignocaine)
B. Nifedipine
C. Digoxin
D. Intravenous isosorbide mononitrate
E. Oral furosemide
30. Un homme de 65 ans avec insuffisance cardiaque a besoin dun traitement pour une fibrillation
auriculaire afin de contrler son rythme cardiaque.
A. Lignocaine (Lidnocaine)
B. Nifidepine

C. Digoxine
D. Mononitrate d'isosorbide en intraveineux
E. Furosemide (Frusemide) per os

31. A 65 year old woman being treated with large doses of loop diuretic requires add-on therapy
for oedema refractory to treatment.
A. Intravenous adenoside
B. Hydrochlorothiazide
C. Oral furosemide
D. Spironolactone
E. Nifedipine
31. Une femme de 65 ans traite avec dimportantes doses de diurtique de Panse a besoin dune
thrapie supplmentaire pour contrler ses dmes rfractaires au traitement

A.
B.
C.
D.
E.

Adenoside par voie intraveineuse


Hydrochlorothiazide
Furosemide (Frusemide) per os
Spironolactone
Nifedipine

32. A 70 year old woman with a history of chronic heart failure presents with severe pulmonary
oedema.
A. 100% oxygen, intravenous diamorphine, intravenous furosemide, sublingual
glyceryl trinitrate (GTN)
B. Digoxin
C. Spironolactine
D. 100% oxygen, oral diamorphine, oral furosemide, intravenous glycerol trinitrate
(GTN)
E. Intravenous adenosine
32. Une femme de 70 ans avec des antcdents dune insuffisance cardiaque chronique se prsente
avec un dme pulmonaire grave.
A. Oxygn 100% Diamorphine en intraveineux, furosemide en intraveineux, trinitrate
glycerole (TNG) en sous-linguale.
B. Digoxine
C. Spironolactone
D. Oxygn 100%, diamorphine per os, furosemide per os, trinitrate glycerol en
intraveineux
E. Adenosine en intraveineux

33. A 56 year old insulin dependent diabetic mellitus (IDDM) male presents with persistently
elevated BP of 180/110mmHg. Blood tests are unremarkable. What is the most appropriate
medication?
A.
B.
C.
D.
E.

Amlodipine
Atenolol
Bendrofluazide
Frusemide (Furosemide)
Perindopril

33. Un diabtique non-insulino-dpendant de 56 ans se prsente avec une tension artrielle


continuellement leve a 180/110mmHg. Les tests sanguins sont non contributifs. Quel est le
mdicament le plus appropri prescrire
A.
B.
C.
D.
E.

Amlodipine
Atenolol
Bendrofluazide
Furosemide (Frusemide)
Pirindopril

instructions: Questions 34-36 :For each clinical scenario below, give the most likely cause for the
clinical findings from the proposed options. / Pour chaque scenario Clinique ci-dessous,
slectionnez la rponse la plus probable parmi les options proposes
34. An anxious 26 year old woman presents with episodes of chest pain and palpitations
precipitated by stress and smoking. Her 24-hour urine shows elevated catecholamines.
A. Cushings syndrome
B. Phaeochromocytoma
C. Hyperparathyroidism
D. Conns syndrome
E. Malignant hypertension
34. Une femme anxieuse de 26 ans se prsente avec des pisodes de douleurs thoraciques et
palpitations prcipits par le stress et le tabagisme. Ses urines de 24 heures montrent un taux lev
des catcholamines.
A.
B.
C.
D.
E.

Syndrome de Cushing
Phaeochromocytome
Hyper parathyrodisme
Syndrome de Conn
Hypertension maligne

35. A 45 year old woman presents with weight gain, muscle weakness and hirsutism. On
examination, she is hypertensive and has pedal oedema.
A.
B.
C.
D.
E.

Cushings syndrome
Phaeochromocytoma
Hyperparathyroidism
Conns syndrome
Renal artery stenosis

35. Une femme de 45 ans se prsente avec gain de poids, faiblesse musculaire et hirsutisme. A
lexamen, elle est hypertensive et prsente des dmes des pieds.
A.
B.
C.
D.
E.

Syndrome de Cushing
Phaeochromocytome
Hyperparathyroidisme
Syndrome de Conn
Stnose de l'artre rnal

36. A 50 year old man presents with right sided hemiparesis. There is a bruit on neck
auscultation. What is the single most appropriate investigation?
A. Blood pressure measurement
B. Carotid Doppler
C. CT scan of head
D. MRI scan
E. Slit-lamp examination
36. Un homme de 50 ans prsente une hmiparsie droite. Il ya un bruit (souffle) lauscultation
du cou. Quelle est la seule investigation la plus approprie
A.
B.
C.
D.
E.

Mesure de la pression artrielle


Doppler carotidienne
Scanner de la tte
IRM
Examen avec lampe fente

InstructionsiOuestions 37-40 :For each case below, choose the single most appropriate diagnosis
from the list of options proposed. / Pour chaque cas dcrit ci-dessous, choisissez le diagnostic le
plus appropri a partir des options proposes.
37. A 52 year-old woman on hormone replacement therapy presents with a swollen left calf, chest
pain and shortness of breath.
A. Pulmonary oedema.
B. Pneumothorax
C. Pulmonary embolism
D. Pneumonia
E. Pleural effusion
37. Une femme de 52 ans sous thrapie de remplacement hormonal se prsente avec un gonflement
du mollet gauche, douleur thoraciqueet essoufflement.
A.
B.
C.
D.
E.

Oedeme pulmonainne
Pneumothorax
Embolie pulmonaire
Pnumonie
Epanchement pleural

38. A 78 year old man has been short of breaih for a few weeks. His chest radiography shows a
right basal shadow rising towards the axilla.
A. Pulmonary oedema
B. Pneumothorax
C. Pneumonia
D. Pleural effusion
E. Pulmonary embolism
38. Un homme de 78 ans a t essouffl depuis quelques semaines. Sa radiographie du thorax
montre une opacit basale droite se dirigeant vers Vaisselle.
A. Oedeme pulmonaire
B. Pneumothorax
C. Pneumonie

D. Epanchement pleural
E. Embolie pulmonaire

39. A 49 year old woman presents with polyuria, ' haemtria, abdominal pain nd bone aches.
On examination her blood pressure is 170/1 OOmmHg.
A. Gallstones
B. Polyarteritis nodosa
C. Pancreatic carcinoma.
D. Chronic pancreatitis
E. Hypercalcaemia
39.Une femme de 49 ans prsente une polyurie, une hmaturiedes douleurs abdominales et
osseuses. A Vexamen sa tension artrielle est iO/lOOmmHg.

A.
B.
C.
D.
E.

Calculs biliaires
Polyarterite noueuse
Carcinome pancratique
Pancratite chronique
Hypercalcmie

40.A 76 year old man fell to the floor while standing in a long queue. He regained consciousness
within 2 minutes. He was not incontinent of urine or stools.
A.
B.
C.
D.
E.

Vaso-vagal syncope
Stokes-Adam attack
Transient ischaemic attack
Carotid sinus syncope
Hypglycaemia

40, Un homme de 76 ans est tomb aprs s'tre tenu dans une longue queue. Il a repris conscience
dans les 2 minutes suivant sa chute. Il n tait pas incontinent pour les selles ou urines.
A.
B.
C.
D.
E.

Syncope vaso-vagal
Sydrome de Stokes-Adam
Attaque ischemique transitoire
Syncope du sinus carutidien
Hypoglycmie
PEDIATRICS /PEPIATRIE

Instructions: Questions 41-46 :For each question below, choose the single most appropriate
answer from the list of options proposed. / Pour chaque question ci-dessous, choisissez la rponse
la plus approprie parmi les options proposes.
41. Which of the following etiologies does not cause cardiac failure in the child 7
A.
B.
C.
D.
E.

Inter ventricular septal defect


Coarctation of the aorta
Fallots tetralogy
Rhythm disorders
Atrio-ventricular septal defect

41. Quelle tiologie nentrane pas une insuffisance cardiaque chez Venfant 7
A.
B.
C.
D.
E.

C.I.V.
Coarctation de laorte
Ttralogie de Fallot
Troubles du rythme
CIAV

42. With respect to the ejection of breast milk, choose the correct statement among the
following propositions.
A. Prolactin provokes the contraction of muscular cells that surround the alveoli
B. The ocytocic reflex is also called milk ejection reflex
C. Ocytocin is secreted more rapidly than prolactin
D. The ejection of milk can be triggered before the child begins suckling
E. During the same suckling feed there can be several ocytoxic reflexes
42. A propos de ljection du lait, choisir ci-dessous la proposition vraie :
A.
B.
C.
D.
E.

La prolactine provoque la contraction des cellules musculaires qui entourent les alvoles
Le rflexe ocytocique est encore appel rflexe djection du lait
L'ocytocine est scrte plus rapidement que la prolactine
Ljection du lait peut tre dclenche avant que l'enfant ne se mette tter
Au cours dune mme tte, il y a plusieurs reflexes ocytociques

43. The following signs are features that can be observed in a case of viral diarrhea except one.
Which one?
A. Notion of an epidemic in a nursery
B. Association with respiratory symptoms
C. Concomittant skin eruption
D. Mucoid and bloody stools
E. Conservation of a good general status
43. Les signes suivants sont retrouvs en cas de diarrhe virale, sauf un : Lequel ?
A.
B.
C.
D.
E.

Notion d'pidmie en crche


Association avec une symptomatologie respiratoire
Eruption concomitante
Selles glairo-sanguinolentes
Conservation de l'tat gnral

44. The Apgar score measures the following parameters except one. Which one?
A. Heart rate of the new born
B. Skin colouration
C. Respiration
D. Chest indrawing (Intercostal indrawing)
E. Tonus
44. Le score dApgar mesure les paramtres suivants lexception dun, lequel ?

A.
B.
C.
D.

Frquence cardiaque du nouveau-n


Coloration cutane
Respiration
Tirage intercostal

E. Tonus
45. In the new-born, which clinical symptom constantly observed can lead to the suspicion of
oesophageal atresia?
A. Cyanosis
B. Respiratory distress
C. Hypersialorrhoea
D. Vomiting
E. Cough
45. Chez le nouveau-n, quel est le symptme clinique constamment retrouv devant faire
souponner le diagnostic datrsie de lsophage ?
A.
B.
C.
D.
E.

Cyanose
Dtresse respiratoire
Hypersialorrhe
Vomissements
Toux

46. A child of 8 years has a recent primary tuberculosis infection. His tuberculin skin test is
positive but he does not present any clinical, radiological or biologic signs. Choose among the
propositions below the single best means of management.
A. Simple observation.
B. BCG vaccination
C. Treatment with rifampicin and isoniazid for 6 months
D. Isoniazid and rifampicin in association with cortico therapy
E. Treatment associating isoniazid, rifampicin and pyrazynamide for 9 months.
46. Un enfant de 8 ans a une primo infection tuberculeuse rcente, il y a un virage des ractions
tuberculiniques, sans signes clinique, radiologique ou biologique. Choisir parmi les propositions cidessous la conduite la plus correcte.
A.
B.
C.
D.
E.

Surveillance simple
Vaccination par BCG
Traitement associant Isoniazide et Rifampicine pendant 6mois
Isoniazide et Rifampicine associs une corticothrapie
Traitement associant isoniazide, Rifampicine et Pyrazinamide pendant 9 mois

Instructions: Questions 47-48 :For each statement below, choose the single most appropriate age
from the list of options proposed. / Pour chaque proposition ci-dessous, slectionnez lge le plus
appropri parmi les options proposes.
47. A child dresses and undresses almost independently
A. 3 years
B. 4 years
C. 5 years
D. 6-8 years
E. 8-10 years
47.Un enfant shabille et se dshabille presque indpendamment

A.
B.
C.
D.
E.

S ans
4 ans
5 ans
6 -8 ans
8-10 ans

48. A child crawls


A. 4-7 months
B. 2-5 months
C. 7-10 months
D. 11-14 months
E. 14-18 months
48. Un enfant marche quatre pattes
A.
B.
C.
D.
E.

4-7 mois
2-5 mois
7-10 mois
11-14 mois
14-18 mois

Instructions: Questions 49-51 :For each case described below, choose the single most appropriate
viral infection from the list of options proposed. / Pour chaque cas dcrit ci-dessous, slectionnez
la rponse la plus approprie parmi les options proposes.
49. A 2 year old boy presents with fever, increased salivation and a swollen left neck.
A. Rubella virus
B. Cosackie virus
C. Mumps virus
D. Cytomegalovirus
E. Measles virus
49. Un garon de 2 ans prsente une fivre, une sialorrhe importante et un gonflement du cou du
cot gauche.

A.
B.
C.
D.
E.

, A. Virus de la rubeole
Virus cosackie
Virus des oreillons
Cytomegalovirus
Virus de la rougeole

50. A 2 year old boy presents with a 12 hour history of fever, cough and hoarseness of voice.
While waiting in the consultation room, he develops stridor.
A. Measles virus
B. Cosackie virus
C. Influenza virus
D. Para influenza virus
E. Rubella virus
50. Un garon de 2 ans prsente avec une fivre, toux et une voix enroue voluant depuis 12
heures. En attendant dans la salle de consultations, il dveloppe un stridor.

A.
B.
C.
D.
E.

Virus de la rougeole
Virus cosacki
Virus de linfluenza
Virus para-influenza
Virus de la rubole

51. A 5 year old child develops severe diarrhoea, nausea and vomiting 1 day after returning from
a school trip.
A. Rubella virus
B. Mumps virus
C. Cosackie virus
D. Hepatitis B virus
E. Rota virus
51. Un enfant de 5 ans dveloppe une diarrhe svre, nause et vomissements un jour aprs le
retour dun voyage dcole.
A.
B.
C.
D.
E.

Virus de la rubole
Virus des oreillons
Virus Cosacki
Virus de lhpatite B
Virus rota (Rota virus)

Instructions: Questions 52-66 : For each case below, choose the single most appropriate
diagnosis from the list of options proposed. / Pour chaque cas ci-dessous, slectionnez le
diagnostic le plus appropri parmi les options proposes.
52. A healthy full term newborn becomes jaundiced at day two. His total bilirubin is 10mg/dl.
His jaundice gradually disappears over the next few days.
A. Hepatitis C
B. Breast milk jaundice
C. Gilberts syndrome
D. Physiological jaundice
E. Galactosaemia
52. Un nouveau-n terme, sain devient ictrique au deuxime jour. Sa bilirubine totale est de
10mg/dl, Son ictre disparait progressivement au cours de quelques jours.
A.
B.
C.
D.
E.

Hpatite C
Ictre du lait maternel
Syndrome de Gilbert
Ictre physiologique
Galactosmie

53. A full term new born becomes jaundiced 14 hours after delivery. His blood results confirm
indirect bilirubinaemia and metabolic acidosis. His parents are both rhesus negative.
A.
B.
C.
D.

ABO incompatibility
Physiological jaundice
Gilberts syndrome
Glucose-6-phosphate dehydrogenase (G6PD) deficiency

E. Crigler-Najjar syndrome
53. Un nouveau n, n terme devient ictrique 14 jours aprs laccouchement. Ses rsultats
sanguins confirment une bilirubinmie indirecte et une acidose mtabolique. Ses parents sont
tous deux rhsus ngatifs.

A.
B.
C.
D.
E.

Incompatibilit ABO
Ictre physiologique
Syndrome de Gilbert
Dficience en GGPD (Glucose-6-phosphate dehydrogenase)
Syndrome de Crigler-Najjar.

54. A male child infertile with testicular atrophy, gynaecomastia and learning difficulties.
A. Edwards syndrome
B. Turners syndrome
C. Downs syndrome
D. Klinefelter* s syndrome
E. Pataus syndrome
54. Un enfant de sexe masculin, striie avec gyncomastie et difficult dapprentissage
A.
B.
C.
D.
E.

Syndrome dEdward
Syndrome de Turner
Syndrome de Down
Syndrome de Klinefelter
Syndrome de Patau

55. Flat face, slanting eyes, simian crease, hypotonia and learning difficulties.
A. Ewards syndrome
B. Turners syndrome
C. Downs syndrome
D. Klinefelters syndrome
E. Triple X syndrome
55. Un enfant prsentant un visage plat, des yeux brids, des plis simians, une hypotonie et des
difficults dapprentissage
A.
B.
C.
D.
E.

Syndrome d'Edward
Syndrome de Turner
Syndrome de Down
Syndrome de Klinefelter
Syndrome de triple X

56. A 3 year old boy with a history of recurrent chest infections presents with worsening
shortness of breath. On examination there was a systolic thrill at the left lower sternal edge,
pansytolic murmur and accentuated second heart sound.
A.
B.
C.
D.
E.

Ventricular septal defect


Atrial septal defect
Patent ductus arteriosus
Fallots tetralogy
Congenital aortic stenosis

56. Un garon de 3 ans aux antcdents dinfections respiratoires rcurrentes se prsente avec une
aggravation dessoufflement. A lexamen, il ya un frmissement systolique au bord infrieur
gauche du sternum, un souffle pansystolique et une accentuation du deuxime bruit cardiaque.
A.
B.
C.
D.
E.

Communication interventriculaire
Communication interauriculaire
Ductus artriosus patent
Ttralogie de Fallot
Stnose aortique congnitale

57. A 12 year old girl presents with marked oedema. Blood tests show hypo-albuminaemia.
Urine shows heavy proteinuria
A. Renal artery stenosis
B. Fanconis syndrome
C. Alport syndrome
D. Renal vein thrombosis
E. Nephrotic syndrome
57. Une fillette de 12 ans prsente des dmes importants. Les tests sanguins montrent une hypoalbuminmie. Lexamen durine montre uneprotinurie massive.
A.
B.
C.
D.
E.

Stnose de lartre rnale


Syndrome de Fanconi
Syndrome dAlport
Thrombose de la veine rnale
Syndrome nphrotique

58. A 3 year old boy presents with a large left-side abdominal mass. Intravenous urography
(IVU) shows a mass within the left kidney which encroaches on the collecting system. Chest
radiography shows multiple pulmonary nodules.
A. Wilms tumour
B. Non-Hodgkins lymphoma
C. Ewings sarcoma
D. Lymphoblastic lymphoma
E. Rhabdomyosarcoma
58.Un garon de 3 ans prsente une grosse masse abdominale du cot gauche. Lurographie
intraveineuse montre une masse au sein du rein gauche qui empite sur le systme collecteur. La
radiographie de thorax montre des nodules pulmonaires multiples.
A.
B.
C.
D.
E.

Tumeur de Wilm
Lymphome non-hodgkinien
Sarcome dEwing
Lymphome lymphoblastique
Rhabdomyosarcome

59. Reduced synthesis of alpha-globin chains leading to haemolysis is called


A. Sickle cell disease
B. Hereditary spherocytosis
C. Alpha thalassaemia

D. Anaemia of chronic disease


E. Microangiopathic haemolytic anaemia
59. Une rduction de synthse des chanes dalpha-globine entranant une hmolyse sappelle:
A.
B.
C.
D.
E.

Drpanocytose
Spherocytose hrditaire
Thalassmie alpha
Anmie des maladies chroniques
Anmie hmolytique micro-angio-pathique

60. A 7 year old boy who presents with night blindness has
A. Scurvy
B. Riboflavin deficiency
C. Beri-beri
D. Vitamin A deficiency
E. Pellagra
60. Un garon de7 ans qui prsente un aveuglement nocturne.
A.
B.
C.
D.
E.

Le scorbut
La Dficience en riboflavine
Le Beri-beri
La dficience en Vitamine 4
Le Pellagra

61. A 14 year old boy presents with a 2 week history of abdominal pain and severe watery
diarrhoea with occasional bright blood mixed with stools. He complains of poor sleep as he has
about 20 motions per day. Examination reveals mild diffuse abdominal tenderness and perioral
skin tags.
A. Ulcerative colitis
B. Crohns disease
C. Meckels diverticulum
D. Hirschsprungs disease
E. Haemorrhoids
61. Un garon de 14 ans prsente une douleur abdominale et une diarrhe liquidienne svre avec
occasionnellement du sang vif mlang avec les selles. Il se plaint dun sommeil pauvre puisquil
fait 20 selles par jour. Lexamen rvle une douleur diffuse lgre la palpation de labdomen et
des plaques pri-orales.
A.
B.
C.
D.
E.

Colite ulcrative
La maladie de Crohn
Diverticule de Meckel
La maladie de Hirschsprung
Hmorrodes

62. A4 year old girl presents with bloody diarrhoea and crampy abdominal pain. Blood tests
show anaemia and thrombocytopenia.
A, Meckels diverticulum
B, Intussusception
C, Haemolytic uraemic syndrome

D, Eosinophilic colitis
E. Hirschsprungs disease

62. Une fille de 4 ans prsente une diarrhe sanglante et de douleurs abdominales type de
crampes. Les tests sanguins montrent une anmie et une thrombocytopnie.
A.
B.
C.
D.
E.

Diverticule de Meckel
Invagination intestinale
Syndrome urmique hmolytique
Colite eosinophile
Maladie dHirschsprung

63. A 6 year old boy presents with mild fever and maculopapular rash. His mother reported that
the rash started on the face then became generalized. On examination, there is palpable cervical
and occipital lymphadenopathy.
A. Measles
B. Rubeila fever
C. Scarlet fever
D. Erythema infectiosum
E. Roseola infantum
63. Un garon de 6 ans prsente une fivre et une ruption cutane maculo-papuleuse. Sa mre a
rapport que lruption a commenc sur le visage, puis est devenue gnralise. A lexamen, il
existe des adnopathies cervicales et occipitales palpabUs.
A.
B.
C.
D.
E.

Rougeole
Rubeole
Scarlatine
Erythema infectiosum
Roseola infantum

64. A9 year old boy presents with conjunctivitis a&d maculopapular rash. The rash started on
the head and spread downwards.
A. Measles
B. Rubella fever
C. Roseola infantum
D. Scarlet fever
E. Pityriasis vesicolor
64. Un garon de 9 ans prsente une conjonctivite et une ruption cutane maculo-papuleuse.
Lruption a commenc sur la tte et stend en descendant
A.
B.
C.
D.
E.

Rougeole
Rubeole
Roseola infantum
Scarlatine
Pityriasis vesicolor

65. A 14 year old girl presents intensely pruritic rash with pustules. On examination the rash is
generalized but is more in the folds between the fingers and toes.

A. Rubella fever
B. Varicella zoster virus (chicken pox)
C. Scabies
D. Scarlet fever
E. Erythema infectiosum
65. Une fille de 14 ans prsente une ruption cutane intensment prurigineuse avec pustules. A
l'examen, lruption est gnralise mais est plus marque dans les plis entre les doigts et les orteils.

A.
B.
C.
D.
E.

Rubeole
Varicella zoster virus (varicelle)
Gale
Scarlatine
Erythema infectiosum

66. A 4 year old boy presents with a vesicular rash that appears in crops.
A. Scabies
B. Measles
C. Rubella fever
D. Varicella zoster virus (chicken pox)
E. Scarlet fever
66. Un garon de 4 ans prsente une ruption vsiculeuse qui apparait en grappe.
A.
B.
C.
D.
E.

La gale
La rougeole
La rubeole
Varicella zoster virus (Varicelle)
Scarlatine

Instructions: Questions 67-71 :For each presentation below, choose the single most likely
diagnosis from the list of options proposed. / Pour chaque prsentation ci-dessous, choisissez le
diagnostic le plus probable parmi les options proposes.
67. A 6 year old girl presents with fever, frequency, dysuria and abdominal pain.
A. Gastro-enteritis
B. Septicaemia
C. Urinary tract infection 1
D. Necrotizing enterocolitis
E. Poisoning
67. Une fille de 6 ans prsente une fivre, une miction frquente, une dysurie et une douleur
abdominale.
A.
B.
C.
D.
E.

Gastr-enterite
Septicmie
Infection urinaire
Entrocolite ncrosante ,
Intoxication (empoisonnement)

68. A 3 week old child presents with projectile vomiting, appearing hungry after each emesis. On
examination dehydration is present and there is a mass in the abdomen.

A. Gastro-oesophageal reflux (physiological)


B. Gastro enteritis
C. Intussusception
D. Pyloric stenosis
E. Gastro-oesophageal reflux (pathological)
68. Un enfant de 3 semaines dge prsente des vomissements en jet, apparaissant affam aprs
chaque vomissement. A Vexamen ta dshydratation est prsente et il ya une masse dans labdomen.

A.
B.
C.
D.
E.

reflux gastro-oesophagien (physiologique)


Gastro-entrite
Invagination intestinale
Stnose du pylore
Refux gastro-oesophagien (pathologique)

69. A 1 year old child presents with vomiting, crying and lifting her legs. Nappy is stained red.
On examination, a sausage shaped mass is palpated per abdominally.
A. Gastro-oesophageal reflux (pathological)
B. Intussusception
C. Pyloric stenosis
D. Gastro enteritis
E. Urinary tract infection
69. Un enfant de 1 an prsente des vomissements et pleure en soulevant les jambes. Les couchesculottes sont taches rouge. A lexamen une masse est palpe dans labdomen
A.
B.
C.
D.
E.

Reflux gastro-oesophagien (pathologique)


Invagination intestinale
Stnose du pylore
Gastro-entrite
Infection urinaire

70. Child with vomiting and diarrhoea. Brother had a .imilar problem 2 weeks ago.
A. Necrotizing enterocolitis
B. Gastro-oesophageal reflux (pathological)
C. Poisoning
D. Septicaemia
E. Gastro-enteritis
70. Un enfant prsente des vomissements et la diarrhe. Le frre avait eu un problme similaire il
ya 2 semaines.
A.
B.
C.
D.
E.

Enterocolite ncrosante
Reflux gastro-oesophagien (pathologique)
Intoxication (empoisonnement)
Septicmie
Gastro-entrite

71. A 14 year old boy complains of sudden pain in his groin after cycling 10 kilometres. What is
the single most likely diagnosis?
A. Femoral hernia
B. Pulled muscle

C. Testicular torsion
D. Urethral trauma
E. Varicocoele
71. Un enfant de 14 ans se plaint de douleurs dinstallation soudaine dans son aine aprs avoir fait
la bicyclette sur une distance de 10 km
A.
B.
C.
D.
E.

Hernie fmorale
tirement musculaire
Torsion testiculaire
Traumatisme urtral
Varicocle

Instructions: Questions 72-73 :For each condition described below, choose the single most
appropriate treatment from the list of options proposed. / Pour chaque condition dcrite cidessous, slectionnez le traitement le plus appropri parmi la liste des options proposes.
72. A 14 year old boy, with well controlled asthma, using inhaled steroids and a bronchodilator
comes to emergency department with breathlessness and swollen lips after eating a peanut,
butter-sandwich.
A. Oral steroids
B. Adrenaline
C. Inhaled sodium cromoglycate
D. Desensitization
E. Inhaled steroids
72. Un garon de 14 ans avec un asthme bien contrl sous corticodes inhals et un
bronchodilatateur vient aux urgences avec essoufflement et un gonflement des lvres aprs avoir
mang un sandwich beurr d'arachides.
A.
B.
C.
D.
E.

Corticodes oraux
Adrnaline
Cromoglycate de sodium inhal
Dsensibilisation
Corticodes inhals

73. A nine year old girl with chronic asthma presents to emergency with rapidly worsening
wheezing not relieved with inhaled bronchodilators.
A. Adrenaline
B. Inhaled sodium cromoglycate
C. Inhaled steroid
D. Intravenous aminophylline
E. Nebulized bronchodilators
73. Une fille de 9 ans avec un asthme chronique se prsente aux urgences avec une respiration
sifflante (wheeze) saggravant rapidement et non soulage par des bronchodilatateurs inhals.
A.
B.
C.
D.
E.

Adrnaline
Cromoglycate de sodium inhal
Corticodes inhals
Aminophylline par voie intraveineuse
Bronchodilatateurs nbuliss

Instructions: Questions 74-75 :For each presentation below, choose the correct answer from the
list of options proposed. / Pour chaque prsentation ci-dessous, slectionnez la rponse juste parmi
la liste des options proposes.
74. Child who is HIV positive is due for measles, mumps and rubella (MMR) vaccine
A. Delay by 2 weeks
B. Give vaccine in hospital
C. Do not give any vaccines
D. Omit measles only
E. Continue as per schedule
74. Un enfant VIH positif doit recevoir les vaccins contre la rougeole, les oreillons et la rubole.
A.
B.
C.
D.
E.

Retarder de 2 semaines
Administrer les vaccins lhpital
Nadministrer aucun vaccin
ter uniquement le vaccin anti-rougeoleux
Continuer comme prvu

75. History of convulsions. Child due for DPT, Haemophilus influenza vaccines etc
A. Delay by 2 weeks
B. Give inactivated vaccines
C. Continue as per schedule
D. Delay by 2 months
E. Omit pertussis only
75. Antcdents de convulsions. Lenfant doit recevoir les vaccins DPT, Haemophilus influenza, etc

A. Retarder de 2 semaines
B. Administrer les vaccins inactivs
C. Continuer comme prvu
D.retarder de 2 mois
E. ter uniquement le vaccin contre la coqueluche
Instructions: Questions 76-78 :For each of the presentations given below, choose the single most
appropriate management from the list of options proposed. / Pour chaque prsentation dcrite cidessous, slectionnez la rponse approprie parmi la liste des options proposes.
76. A young boy with polyuria, has very high blood glucose levels
Insulin only
Insulin with normal saline
Normal saline with potassium
Normal saline with hydrocortisone
Saline and potassium supplementation
76. Un jeune garon avec polyurie a des taux de glycmie trs levs
A.
B.
C.
D.
E.

A.
B.
C.
D.
E.

Insuline seule uniquement


Insuline avec srum sal normal
Srum sal normal avec du potassium
Srum sal avec hydrocortisone
Srum sal avec supplment de potassium

77. A 1 year old child develops swollen lips after eating peanuts. What is the single most
appropriate management?
Administer intra-muscular adrenaline
Administer intravenous hydrocortisone
Administer intravenous chlorphenamine
Admit to hospital urgently
Prescribe oral antihistamine
77. Un enfant de 1 an dveloppe des lvres gonfles aprs avoir mang des arachides
A.
B.
C.
D.
E.

A.
B.
C.
D.
E.

Administrer ladrnaline par voie intra-musculaire


Administrer lhydrocortisone par voie intraveineuse
Administrer la chlorphenamine par voie intraveineuse
Faire admettre lhpital en urgence
Prescrire lantihistaminique par voie orale

78. A 10 year old child presents with 15% body surface area burns. What is the single most
appropriate management?
A. Administer intravenous fluids, clean, dress wounds in emergency and discharge.
B. Admit, notify social services, puncture blisters and perform escharotomy as needed
C. Irrigate with cold water, dress wounds and discharge with silver sulfadiazine and
dressings
D. Clean, use dry occlusive dressings and discharge on broad spectrum antibiotics.
E. Refer to specialist burns unit
78. Un enfant de 10 ans prsente des brulures couvrant 15% de sa surface corporelle.
A. Administrer les soluts par voie intraveineuse, nettoyer et faire le pansement des plaies aux
urgences et faire sortir de lhpital.
B. Hospitalisation, notifier les services sociaux, ponctionner les phlyctnes et faire une
escarotomie selon les besoins.
C. Irriguer avec Veau froide, faire le pansement des plaies et faire sortir de lhpital avec
sulfadiazine d'argent et des compresses.
D. Nettoyer, utiliser le pansement occlusif sec et faire sortir de lhpital avec des antibiotiques
large spectre
E. Rfrer au service spcialis pour des brls

Instructions: Questions 79-80 :For each of the presentations given below, choose the single most
likely mechanism of the disease from the list of options proposed. I Pour chaque prsentation
dcrite ci-dessous, slectionnez la rponse la plus appropries parmi les options proposes.
79. Bronchiolitis
A. Viral infection
B. Surfactant deficiency
C. Ciliary dyskinesia
D. Allergy
E. Immune process
79. Bronchiolite
A. Infection virale
B. Dficience en surfactant
C. Dyskinsie

D. Allergie
E. Processus immune

80. A boy with mental retardation and learning difficulties. Father and uncle had similar problems.
A. Alpha-1 antitrypsin deficiency
B. ciliary dyskinesia
C. genetic
D. immune process
E. trinucleotide repeats
80. un garon avec retard et des difficults dapprentissage. Le pre et loncle avaient des
problmes Similaires.
A.
B.
C.
D.
E.

Dficience en alpha 1 antitrypsine


Dyskinsie ciliaire
Genetique
Processus immun
Reprise trinucleotide (trinucleotid repeats)
PUBLIC HEALTH / SANTE PUBLIQUE

Instructions : Questions 81-92 : Choose the correct answer from the following propositions /
Choisir la bonne rponse parmi les propositions suivantes.
81. Dialogue structures
A. Exist at all the levels of the health system pyramid
B. Does not concern confessional health facilities
C. Are not necessary in case of conflict
D. Are imposed on all countries by the World Bank
E. Are an initiative of Dr. Eugene Jamot
81. Les structures de Dialogue

A.
B.
C.
D.
E.

Existent tous les niveaux del pyramide sanitaire


Ne concernent pas les structures sanitaires confessionnelles
Ne sont pas ncessaires en cas de conflits
Sont imposes tous les pays par la Banque Mondiale
Sont une initiative du Docteur Eugene Jamot

82. Concerning the functioning of dialogue structures


A. The head of the health facility presides over the health committee and health management
committee is presided over by a member of the community.
B. The chairperson of the health management team can be a civil servant working at an integrated
health centre
C. The secretariat of the health committee and that of the health management committee cannot be
held at the same time by the head of the health facility
D. The funds managed by the health management committee are public funds
E. The senior divisional officer can preside the district health committee in case of conflict with
members of the community
82. Concernant le fonctionnement des structures de dialogue

A. Le chef de la formation sanitaire prsid le COSA'st le COGE est prsid pat un membre de la
communaut
B. Le prsident du COGE, peut tre un fonctionnaire du Centre de sant intgr
C. Le secrtariat du COSA et celui du COGE, ne peuvent pas tre tenus en mme temps par le chef
de la formation sanitaire
D. Les fonds grs par le COGE sont des fonds publics
E. Le prfet peut prsider le COSADI en cas de querelle des membres de la communaut

83. Concerning the functioning of the dialogue structure


A. The district head is a member of the district health committee
B. The district health committee has competence over the whole district contrarily to that of the
district health management committee.
C. The bureau of the district health committee is composed solely of chair persons of health
management committees or health committees of the health areas
D. The council is officially a dialogue structure of the health system
E. All the above propositions are false
83. Concernant le fonctionnement des structures de dialogue
A. Le Chef du district est membre du COSADI
B. La comptence du, COSADI stend tout le District .de sant, contrairement celle du COGEDI ;
C. Le bureau du COSADI est compos uniquement des prsidents des COGE ou des COSA des aires
D. La Mairie est officiellement une structure de dialogue dans le systme de sant
E. Toutes les propositions prcdentes sont fausses
84. Concerning the organization of health district
A.
B.
C.
D.
E.

A health district can be situated across two regions


A health district can be situated across two divisions
The district hospital can never be a confessional (missionary) hospital
A private health centre can never be responsible for a health area
Considering the population density in big towns and the necessary minimal distances, a health
district can dispose of only one health area.
84. Organisation du district de sant
A. Un district de sant peut tre cheval sur deux Rgions
B. Un district de sant peut tre cheval sur deux dpartements
C. Lhpital de district ne peut jamais tre un hpital confessionnel.
D. Un centre de sant priv ne peut jamais tre responsable dune aire de sant
E. Vues la concentration des populations dans les grandes villes, et les distances minimales
ncessaires, un district de sant peut disposer dune seule aire de sant
85. Suppose that the blood sugar level is approximatively normal for a group of women whose
standard deviation is unknown. What is the appropriate test to compare the means?
A. Students t-test
B. Z-test
C. Chi2
D. Fishers exact test
E. MacNemars test
85.On suppose que le taux de glycmie est approximativement normal pour un groupe de femmes
avec cart-type inconnu. Quel est le test appropri pour comparer les moyennes?
A. Test de Student
B. Test Z del cart rduit
C. Chi-deux

D. Test exact de Fisher


E. Test de MacNernar
86. To test an association between tobacco use (Yes/No) and pneumonia (Yes/No), a Chi square
value of 2.96 was found. What decision will you take?
A. There is no significant association between tobacco use and pneumonia
B. There is a significant association between tobacco use and pneumonia
C. There is an association between tobacco use and pneumonia
D. There is no association between tobacco use and pneumonia
E. The statement cannot permit to take a decision
86. Pour tester ( 5%) une association entre le tabac (oui/non) et la pneumonie (oui/non), on a
trouv une valeur de chi-deux calcule gale 2.96. Quelle dcision prendre ?
A.
B.
C.
D.
E.

Il n existe aucune association significative entre le tabac et la pneumonie


Il existe une association significative entre le tabac et la pneumonie
Il existe une association entre le tabac et la pneumonie
Il n existe aucune association entre le tabac et la pneumonie
Lnonc ne peut pas permettre de prendre une dcision

87. In a bid to compare at 5% the CD4 cell counts between 20 men and 22 women, a p-value of
0.06 was found. What conclusion can be drawn?
A. There is no significant difference between the CD4 cell counts because 0.06>0.05
B. There is a significant difference between the CD4 cell counts because 0.06>0.05
C. There is no significant difference between the CD4 cell counts because the sample size is small
D. There is a significant difference between the CD4 cell counts because the sample size is
E. None of the above
87.Dans le but de comparer 5%, les taux de CB4 entre 20 hommes et 22 femmes, on a trouv une
valeur-p de 0,06, Quelle conclusion peut-on tirer?
A.
B.
C.
D.
E.

Il existe une diffrence non significative entre les taux de CD4 car 0,06 >0,05
Il existe une diffrence significative de taux de CD4 car 0.06 >0.05
Il existe une diffrence non significative de taux de CD4 car taille dchantillon faible
Il existe une diffrence significative de taux de CD4 car taille dchantillon faible
Aucune des prcdentes

88. Which set of the three elements constitutes characteristics of centrai tendency?
A. Standard deviation, mean and mode
B. Mean, median and mode
C. Mean, quartile and median
D. Mean, spread and standard deviation
E. Median, mean and standard deviation
88. Quel triplet parmi les suivants constitue les caractristiques de tendance Centrale?
A. Ecart-tye, moyenne et mode
B. Moyenne, mdiane et mode
C. Moyenne, quartile, et mdiane
D. Moyenne, tendu et cart-type
E. Mdiane, Moyenne et cart-type
89. We want to test whether there is an association between obesity (1, II, HI) and religion. The
number of subjects in one of the cells is 2.4, which test is the most appropriate?
A. Fishers exact test

B. MacNemars test
C. Chi2 for trend
D. Adjusted Chi2 test
E. Homogeneity Chi2 test
89.On veut tester s'il y a une association entre lobsit (I, II et III) et la confession religieuse,
leffectif attendu dans une case est de 2,4. Quel test est le plus appropri ?

A.
B.
C.
D.
E.

Test exacte de Fisher


Test de MacNemar
Test de Chi-deux de tendance
Test de Chi-deux dajustement
Test de Chi-deux d'homognit

90. In which case is the Chi2 test and variant not applicable?
A. Continuous variables ,
B. Binary variable
C. Categorical variables
D. Discrete variables
E. All of the above
90. Dans quel cas le test de chi-deux et variantes ne peuvent pas s appliquer :
A. Variables continues
B. Variables binaires
C. Variables catgorielles
D. Variables discrtes
E. Toutes les prcdentes

91. Which set of pairs below always goes together?


A. Interquartile interval and mean
B. Mean and standard deviation
C. Standard deviation and interquartile
D. Median and standard deviation
E. All of the above
91.Quelle est la paire qui va toujours ensemble ?
A.
B.
C.
D.
E.

Intervalle interquartile et moyenne


Moyenne et cart-type
Ecart-type et intervalle interquartile
Mdiane et cart-type
Toutes les prcdentes

92. In which circumstance is it necessary to use non-parametric tests?


A. Comparison of means
B. Data distribution known in advance
C. Comparison of proportions
D. Large sample size
E. Normal distribution of data
92. Dans quelle circonstance est-il ncessaire d'utiliser les tests non-paramtriques?
A. Comparaison des moyennes
B. Distribution des donnes connue davance
C. Comparaison de proportions

D. Taille dchantillon trs grande


E. Distribution normale de donnes.
Questions 93-100:

A group of male workers between the ages of 20 and 39 years are being screened for lung disease
by spirometry. Nine subjects are examined. Their forced expiratory volume in 1 second (FEV1)
divided by forced vital capacity (FEV1/FVC%) results are 80, 76, 73, 61, 64, 79, 64, 64, and 78./
Un groupe de travailleurs gs entre 20 et 39 ans suivent un dpistage pour un cancer du poumon
par spiromtrie. Neuf sujets sont examines. Leur VEMS est divise par la capacit vitale force
(VEMS/FVC%) et les rsultats sont : 80, 76, 73, 61, 64, 79, 64, 64 et 78.
93. What is the modalreading?
A. 61
B. 64
C. 71
D. 73
E. 76
93. Quel est le mode?
A. 61
B. 64
C. 71
D. 73
E. 76

94. What is the median value?


A. 61
B. 64
C. 71
D. 73
E. 76
94.Quel est la valeur mdiane?
A. 61
B. 64
C. 71
D. 73
E. 76
95. What is the range?
A. 17
B. 18
C. 19
D. 20
E. 21
95. Quelle est lextreme (range)?
A. 17
B. 18
C. 19
D. 20
E. 21
96. A new screening test for prostate cancer becomes available. You assess whether this will be
useful to your practice by reviewing the operating characteristics of the test. The tests
ability to correctly classify diseased persons as having disease is called?

A.
B.
C.
D.
E.

Specificity
Sensitivity
Positive predictive value
Negative predictive value
Reproducibility

96. Un nouveau test pour le dpistage du cancer de la prostate devient disponible. Vous valuez pour
savoir sil sera utile pour votre travail en examinant ses caractristiques opratoires. La capacit
du test correctement classer des personnes malades comme ayant la maladie sappelle:
A. Spcificit
B. Sensitivit
C. Valeur prdictive positive
D. Valeur prdictive ngative
E. Reproductibilit
97. The laboratory test has a sensitivity of 85% and a specificity of 70%. You want to have 1000
persons take the test. To estimate the positive predictive value of the test for this population,
you need to know which of the following?
A. Disease incidence
B. Disease prevalence
C. Negative predictive value
D. The cut off values for the test
E. The latent period of the disease
97. Un test de laboratoire a une sensitivit de 85% et une spcificit de 70%. Vous voulez faire le test
1000personnes. Pour valuer la valeur prdictive positive de ce test pour cette population. Vous
devez connatre lequel des lments suivant?
A. Lincidence de la maladie
B. La prvalence de la maladie
C. La valeur prdictive ngative
D. Les valeurs seuils pour le test
E. La priode latente de la maladie
98. Among the observational study designs, the prospective cohort design offers which one of the
following advantages?
A. It provides a relatively quick answer.
B. It allows one to take advantage of existing outcome data.
C. It is easy to assemble a comparison group.
D. It allows one to measure incidence.
E. It is relatively cheap.
98. Parmi les schmas (designs) des tudes observationnelles, la cohorte prospective offre lequel des
avantages suivants?
A. Elle permet d'avoir une rponse relativement rapide
>
B. Elle permet de prendre lavantage des donnes existantes sur les rsultats (devenir)
C. Il est facile de rassembler un groupe de comparaison
D. Elle permet de mesurer l'incidence
E. Elle est relativement pas chre ou bon march
99. Which of the following is not a health programme in Cameroon?
A. Expanded Programme of Immunization (EPI)
B. TB Control
C. Malaria Control
D. Control of Neglected Tropical Diseases
E. Control of Intestinal Worms

99. Laquelle des propositions suivantes nest pas un programme de sant au Cameroun?
A. Programme largi de vaccination (PEV)
B. Lutte contre la tuberculose
C. Lutte contre le paludisme
D. Lutte contre les maladies orphelines
E. Lutte contre les vers intestinaux
100.

In Cameroon Health System the district hospital is considered as


A. The locus of integrated continuous and comprehensive health care
B. The first referral level
C. The only unit of first contact with the community
D. The second referral level'
E. None of the above
100. Dans le systme de sant au Cameroun, hpital de district est considr comme
A.
B.
C.
D.
E.

Un site des soins de sant intgr continu et complet


Le premier niveau de rfrence
La seule unit de premier contact avec la communaut
Le deuxime niveau de rfrence
Aucune de ces rponses nest juste

END /FIN

RPUBLIQUE DU CAMEROUN

REPUBLIC OF CAMEROUN

Paix-Travail-Patrie

Peace - Work - Fatherland

MINISTERE DE L'ENSEIGNEMENT SUPERIEUR

MINISTRY OF HIGHER EDUCATION

COMMISSION NATIONALE DE LA FORMATION


EXAMEN NATIONAL DE SYNTHSE CLINIQUE ET THRAPEUTIQUE DE LA FORMATION
MDICALE AU CAMEROUN
NATIONAL COMPREHENSIVE CLINICAL AND THERAPEUTIC EXAMINATION FOR
MEDICALTRAINING IN CAMEROON
2nde PARTIE
EPREUVES DE CHIRURGIE ET SOUS SPECIALITES, GYNECOLOGIE- OBSTETRIQUE ET
ANESTHESIE, Dure : 3heures
2ndPART
EXAMINATION OF SURGERY, GYNAECOLOGY-OBSTETRICS AND
ANESTHESIA, Duration; 3hours
12 Septembre 2014
CAHIER DE QUESTIONS
NOM
NAME
PRENOM
N RECEPISSE RECEIPT
N
N DE TABLE SERIES N
DATE

SIGNATURE

QUESTION BOKLET
CE CAHIER COMPORTE 100 QUESTIONS. LES REPONSES DOIVENT ETRE ECRITES
UNIQUEMENT SUR LA FEUILLE-REPONSE. CE CAHIER DOIT ETRE REMIS A LA FIN DE
LEPREUVE AVEC LA FEUILLE-REPONSE.
THIS MCQ IS MADE OF 100 QUESTIONS. ALL YOUR ANSWERS MUST BE WRITTEN ONLY ON THE
ANSWER SHEET.
THIS BOOKLET SHALL BE GIVEN BACK WITH THE ANSWER SHEET.

Instructions : Questions 1-12 : Choose the correct answer from the following propositions /
Choisir la bonne rponse parmi les propositions suivantes.
1. Among the following signs which one can be considered as being almost pathognomonic of
evolved chronic pancreatitis
F. Stealorrhoea 10g/24 hours
G. Diffuse increase in the density of pancreatic parenchyma on the CT scan
H. Presence of calcifications on the pancreatic area seen on a plain abdominal X-ray
I. Dilatation of Wirsung canal on echography (ultrsonography)
J. Short stenosis of the main biliary canal on retrograde cathe terisaiion of the papilla
1.
Parmi les signes suivants quel est celui considr comme quasi-pathognomonique dune
pancratique chronique volue ?
F. Statorrhe 10g/24heures
G. Augmentation diffuse de la densit du parenchyme pancratique au scanner
H. Prsence de calcifications dans laire pancratique visible sur la radio sans prparation de
l'abdomen
I. Dilatation du canal de wirsung a Vchographie
J. Stnose courte de la voie biliaire principale lors du cathtrisme rtrograde de la papille.
2.
In the list below, which one is incompatible with the diagnosis of intra-epithelial
carcinoma of the cervix?
a. Abnormal mitoses
b. Architectural disorganization
c. Important atypical nucli
d. Inflammatory infiltration of the underlying chorain
e. Lymphatic emboli
2. Quel est dans la liste des lments ci-aprs celui qui est incompatible avec un diagnostic de
carcinome intra-pithlial du col utrin?
a. Mitoses anormales
b. Dsorganisation architecturale
c. Importantes atypies nuclaires
d. Infiltrt inflammatoire du chorion sous -jacent
e. Embolies lymphatiques

3. In the observation o a patient with a fracture of the vault of the skull associated with a brief
loss of consciousness, the following features point to an extra-durai hematoma except one.
Which one?
a. Worsening of the state of consciousness after the 4 th hour
b. Notion of a temporal fracture
c. Immediate deep coma
d. Unilateral mydriasis
e. Hemeplegia of late onset
3. Dans la surveillance dune fracture de la vote crnienne, avec perte de connaissance brve, des
lments ci-dessous font penser un hmatome extradural,sauf un. Lequel?
a.
b.
c.
d.
e.

Aggravation de ltat de conscience aprs la 4eme heure


Notion dun trait de fracture temporal *
Coma grave demble
Mydriase unilatrale
Hmiplgie dinstallation retarde

4. Among the following cancers, which one has an increase in incidence due to the inhalation of
asbestos fibres?
A. Cancer of the bladder B. Cancer of the oesophagus
C. Cancer of the Stomach
D. Cancer of the pleura
E. Cancer of the ethmoid
4. Parmi les cancers suivants, quel est celui dont lincidence augmente avec linhalation de fibres
damiante ?
A. Cancer de la vessie
B. Cancer de lsophage
C. Cancer de lestomac
D. Cancer de la plvre
E. Cancer de lethmode
5. Among the following propositions which one is false concerning pseudo-arthrosis after a
fracture of the neck of the femur?
A. Is favoured by a vertical fracture
B. Is more common after a fracture of the region of the trochanter than after a true cervical
fracture
C. Can be accompanied by necrosis of the femoral head
D. Is usually treated in the elderly by a prothesis ,
E. Is treated in youngsters by osteo-synthesis
5. Concernant la pseudarthrose aprs fracture du col du fmur, parmi les propositions suivantes
indiquez celle qui est fausse :
A. Est favorise par un trait de fracture vertical
B. Est plus frquente aprs fracture de la rgion trochantrienne qu 'aprs fracture cervicale vraie
C. Peut saccompagner de ncrose de la tte fmorale
D. Est habituellement traite chez le vieillard par la mise en place d une prothse
E. Est traite chez le jeune par une ostosynthse.
6. A surgical operation is indicated for a prostatic adenoma when
A. It is voluminous
B. There is frank dysuria
C. There is impotence
D. The subject is not in renal failure
E. There is hematuria
6. Un adnome de la prostate devient une indication opratoire :
A. Il est trs volumineux
B. Il existe une dysurie franche
C. Il existe une impuissance
D. Le sujet n est pas insuffisant rnal
E. Il existe une hmaturie
7. Among the following complications of chronic pancreatitis which one is false?
A. Stealorrhoea
B. Diabetes
C. Pseudo-cyst of the pancreas

D. Constipation
E. Intestinal haemorrhage due to segmentary portal hypertension
7. Parmi les complications suivantes de la pancratite chronique, une seule est fausse. Laquelle?
A. Statorrhe
B. Diabte
C. Faux kyste du pancras
D. Constipation
E. Hmorragies digestives par hypertension portale segmentaire
8. Among the following clinical signs, one is not a component of a complete acute ischaemic
syndrome of a limb observed in the first six hours. Which one?
A. Decrease in local temperature
B. Disappearance of distal pulses
C. Malleolar oedema
D. Sensitivo-motor paralysis
E. Gangrene of the toe
8. Parmi les signes cliniques suivants, un ne fait pas partie du syndrome dischmie aigiie complte
dun membre vu dans les 6 premires heures. Lequel?
A. Diminution de la chaleur locale
B. Abolition de pouls distaux
C. dme mallolaire
D. Paralysie sensitivomotrice
E. Gangrne dun orteil
9. Patients who have had a splenectomy are exposed to severe bacterial septicaemia. Among the
following bacteria, which one particularly threatens these subjects?
A. Staphylococus
B. Colibacillus
C. Samonella
D. Pneumococeus
E. Listeria
9. Les splnectomiss sont exposs des septicmies bactriennes gravissimes. Parmi les bactries
suivantes, quelle espce menace particulirement ces sujets ?
A. Staphylocoque dore
B. Colibacille
C.Salmonelle
D.Pneumocoque
E. Listeria

F.
G.
H.
I.
J.

10. In case of cancer of the head of the pancreas, the following signs can be observed except one.
Which one?
Progressive jaundice
Severe weight loss
Fever with rigors
Pain in the form of liver colic '
Megacholecyst (Increase in the volume of gall bladder).
10 En cas de cancer de la tte du pancras, tous les signes peuvent tre observs sauf un. Lequel ?
A. Ictre progressif

B.
C.
D.
E.

Amaigrissement important
Fivre avec frisson
Douleur type de colique hpatique
Mgacholcyste (augmentation du volume de la vsicule biliaire)

11 The following surgical diseases constitute differential diagnoses of intestinal obstruction the
except one. Indicate the one that is false
A. Strangulated hernia
B. Abdominal contusion
C. Acute pancreatitis
D. Ileo-mesenteric infarction
E. Paralytic ilus
11. Les affections chirurgicales suivantes font partie des diagnostics diffrentiels de locclusion
intestinale ; indiquez celle qui est fausse
A.
B.
C.
D.
E.

Hernie trangle
Contusion abdominale
Pancratique aigu
Infarctus ilo msentrique
Ilus paralytique

12 Intestinal obstruction of the elderly leads to the suspicion of the following etiologies except
one. Which one?
A. Obstruction of the colon due to cancer
B. Hirschsprungs disease (Megacolon)
C. Obstruction of the small intestines by biliary ileus
D. Olgives syndrome
E. Pseudo-obstruction of the colon by a fecalome
12. L occlusion intestinale du sujet g fait penser aux tiologies suivantes sauf une. Laquelle ?
A. Occlusion colique sur cancer
B. Maladie de HIRSCHSPRUNG (mgaclon)
C. Occlusion du grle par ilus biliaire
D. Syndrome dOLGIVIE
E. Pseudo-obstruction colique par fcalomes
Instructions: Questions 13-18 : For each question, choose th correct answer from the different
propositions / Pour chaque question, choisissez la rponse correcte parmi tes diffrentes
propositions.
13. Among the following statements, which one relates to common bile duct stones:
A. Most are formed primarily in the common bile duct
B. Most are associated with an elevated serum alkaline phosphatase
C. About 50% are asymptomatic
D. They are often associated with a palpable gall bladder
E. They cause at least one third of all episodes of pancreatitis
13. Parmi'les propositions suivantes, laquelle est en rapport avec les calculs de la voie biliaire
commune?
A. La plupart sont formes principalement dans la voie biliaire commune
B. La plupart sont associes un taux lev des phosphatases alcalines
C. Environ 50% sont asymptomatique
D. Ils sont souvent associs une vsicule biliaire palpable

E. Ils sont responsables pour au moins un tiers dpisodes de pancratite


14. Crohns disease
A. Rarely fstulates
B. May present as recurrent perianal abscess
C. Can involve the mouth
D. Is characterized by cseating epitheloid granulomas
E. Occasionally causes a fulminant colitis
14. La maladie de Crohn
A.
B.
C.
D.
E.

Se fistule rarement
Peut se prsenter comme un abcs pri-anal rcidivant
Peut affecter la bouch
Est caractrise par un granulome pithloide caseux
Provoque occasionnellement une colite fulminante

15. In patients with upper gastro-intestinal bleeding


A. The bleeding point is frequently beyond the ligament of Treitz
B. Angiodysplastic lesions are a common cause
C. The patients always present with haematemesis
D. Diagnostic endoscopy is best done within the first 24 hours
E. Selective mesenterjc angiography is the key diagnostic step.
15. Chez les patients prsentant une hmorragie gastro-intestinale haute
A. Le point de saignement est frquemment situ au del du ligament de Treitz
B. Les lsions angiodysplasiques sont la cause la plus frquente
C. Les patients se prservent toujours avec une hmatmse
D. Lendoscopie diagnostique est mieux faite dans les 24 premires heures
E. Angiographie msentrique slectiveest ltape cl du diagnostic
16. 25 year old man presents with a blue right arm with absent radial pulse and painful passive
finger extension after a supracondylar fracture of the humerus. He has:
A. A compartment syndrome
B. An ulnar nerve injury
C. A median nerve injury
D. A radial nerve injury
E. None of the above
16. Un homme de 25 ans prsente un bras droit bleu avec Vabsence du pouls radial et une extension
passive des doigts douloureuse aprs une fracture supra-condylienne de lhumrus. Il a
A. Un syndrome de compartiment
B. Une lsion du nerf ulnaire
C. Une lsion du nerf mdian
D. Une lsion du nerf radial
E. Aucune de ces propositions nest vraie
17. A 70 year old woman complains of right sided pleuritic chest pain 10 days after a fractured
neck of the femur.
E, Compartment syndrome
F, Sudecks atropy or complex regional pain syndrome type 1
G, Pulmonary embolus
H, Pneumothorax
I, None of the above

17. Une femme de 70 ans se plaint dune douleur de type pleural hmithoracique droit dix jours
aprs une fracture du col du fmur.
A.
B.
C.
D.
E.

Un syndrome de compartiment
Atrophie de Sudeck ou syndrome de douleur rgionale complexe de type 1
Embolie pulmonaire ,
Pneumothorax
Aucune de ces propositions n 'est vraie

18. A 65 year old woman who falls on an out-stretched arm shows weakness of wrist extensors.
She has:
A. An ulnar nrve injury
B. A median nerva injury
C. A radial nerve injury
D. A Sudecks atropy
E. None of the above
18.Une femme de 65 ans fait une chute sur son bras en extension et prsente une faiblesse des
muscles extenseurs du poignet. Elle a
A. Une lsion du nerf ulnaire
B. Une lsion du nerf mdian
C. Une lsion du nerf radial
D. Une atrophie de Sudeck
E. Aucune de ces propositions nest vraie
Instructions: Questions 19-21 : For each clinical scenario below, suggest the most appropriate
management from the options proposed. / Pour chaque scnario clinique ci-dessous, choisissez la
rponse la plus approprie parmi les options proposes.
19. A 35 year old woman presents after a fall on an outstretched arm. Her radiograph shows an
undisplaced transverse fracture of the right humerus.
A. Broad sling
B. Collar-and-cuff sling
C. Internal fixation
D. Rest and mobilization
E. None of the above
19.Une femme de 35 ans se prsente aprs une chute sur son bras en extension. Sa radiographie
montre une fracture transverse non-dplace de lhumrus droit
A.
B.
C.
D.
E.

Echarpe large
Collier et charpe au poignet
Fixation interne
Repos et mobilisation
Aucune des propositions n est vraie

20. A 24 year old man presents with a swollen painful hand after falling over playing squash. His
radiograph reveals a scaphoid fractur.
A. Plaster from wrist to above knuckle
B. 'Plaster from below elbow to above knuckle
C. Manipulation under local anaesthesia and cast
D. Rest and mobilization
E. Internal fixation

20.Un homme de 24 ans prsente une main enfle et douloureuse aprs une chute en jouant au squash.
A. Placer un pltre du poignet jusqu 'au dessus de larticulation
B. Placer un pltre au dessus du coude jusqu au dessus de larticulation
C. Faire une manipulation sous anesthsie locale et placer un pltre
D. Repos et mobilisation
E. Fixation interne.
21. A 65 year old woman requires treatment for displaced fractured neck of femur.
A. Gallows traction
B. Manipulation under local anaesthesia and cast
C. Rest and mobilisation .
D. Internal fixation
E. Bone traction
21.Une femme de 65 ans a besoin des soins aprs une fracture dplace du col du fmur
A.
B.
C.
D.
E.

Traction de Gallow
Manipulation sous anesthsie locale et pose de pltre
Repos et mobilisation
Fixation interne
Traction de los

Instructions; Questions 22-50 : Choose the best answer from the following propositions /
Choisissez parmi les propositions suivantes la rponse la plus exacte.
22. Which of the following statements concerning fluid and electrolyte balance are correct?
A. Nasogastric aspirates should be replaced volume for volume with 5% dextrose solution.
B. 100 mmol of potassium are required each day to replace baseline losses.
C. Long standing fluid deficits should be replaced within the first 24 hours
D. Insensible losses are unchanged by fever.
.
E. A total daily intravenous fluid intake of 2500ml will maintain baseline fluid requirements in a
normal individual.
22.Parmi les propositions suivantes concernant lquilibre hydro-lectrolytique, laquelle est exacte?
A. Le liquide obtenu par aspiration naso-gastrique doit tre remplac volume pour volume par
une solution glucose 5%
B. 100 mmol du potassium sont ncessaires chaque jour pour remplacer les pertes de base
C. Les dficits au long cours doivent tre remplacs dam les 24 premires heures.
D. Les pertes insensibles ne sent pas modifies par la fivre
E. La prise totale quotidienne de 2500ml des liquides administrs par voie intraveineuse
maintiendra ltat de base les besoins liquidiens.
23. A deteriorating level of consciousness in the accident department an hour after serious head
injury
A. Is almost certainly caused by an expanding intracranial haematoma
B. May reflect rising intracranial pressure
C. Should routinely be treated with mannitol and steroids. ,
D. Is not significant unless the patient also develops focal neurological signs
E. Automatically requires immediate transfer to th nearest to the nearest CT scanner
23. Une dtrioration du niveau de conscience aux urgences une heure aprs un traumatisme
crnien svre.
A. Est presque certainement lie a un hmatome intracrnien en expansio

B.
C.
D.
E.

Peut reflter une lvation de la pression intracrnienne


Doit de faon mutinire tre traite par mannitol et corticostrodes
Nest pas importante sauf si le patient dveloppe aussi des signes neurologiques focaux
Ncessite automatiquement un transfert immdiat dans un centre de scanner la plus proche

24. A 28-yr old man presents with recurrent Crohns disease. He has previously had multiple
small bowel resections and has 150cm of small intestine left. On this occasion he has been
admitted with an enterocutaneous fistula. Which of the following options would provide the most
suitable nutritional support?
A. Naso-jejunal tube feeding
B. Parenteral nutrition via a standard grey cannula
C. A feeding jejunostomy
D. Oral nutrition delivered via supplementation using high-protein-calorie drinks
E. Parenteral through a tunneled central venous catheter
24. Un homme de 24 ans se prsente avec une maladie de Crohn rcidivante. Il a dj eu
auparavant des rsections multiples de lintestin grle et conserve actuellement 150cm de
lintestin grle. Actuellement, il a t admis pour une fistule antro-cutane. Laquelle des
propositions suivantes donnera un soutien nutritionnel le plus approprie
A. Alimentation par sonde naso-jejunale
B. Alimentation parentrale par une canule gris standard
C. Alimentation travers la jejunostomie
D. Alimentation orale dlivre la supplmentation par des boissons riches en protines et
calories
E. Alimentation par voie parentale travers un catheter veineux central
25. Which of the following statements relate to peptic ulcer disease:
A. Ulcers are most reliably detected by a barium meal
B. Pyloric stenosis is the least frequent complication
C. Elective surgery is usually required
D. A drainage procedure is not always required after truncal vagotomy
E. Radiological evidence of subdiagphragmatic gas is always present after peptic ulcer
perforation.
25.Laquelle de ces propositions est en rapport avec la maladie ulcreuse peptique?
A.
B.
C.
D.
E.

Les ulcres sont dtects de faon fiable par un transit baryt


La stnose du pylore est la complication la moins frquente
La chirurgie programme est habituellement ncessaire ou requise.
Une procdure de drainage n est pas toujours ncessaire aprs vagotomie troncale
La mise en vidence radiologique de gaz sous diaphragmatique est toujours prsente aprs
perforation d'un ulcrepeptique.

26. Necrotising pancreatitis:


A. Complicates half of all episodes of acute pancreatitis
B. Is not associated with a poor outcome
C. Can be diagnosed on ultrasound scanning
D. Responds to radical surgical debridement
E. Is usually sterile
27. La pancratite ncrosante
A. Complique la moiti de toutes les pisodes de pancratites aiges
B. N'est pas associe avec un mauvais devenir
C. Peut tre diagnostique par me chographie

D. Rpond bien au dbridenient chirurgical radial


E. Est habituellement strile
27. Which of the fpllowing is the most common cause of obstructive jaundice in patients with
chronic pancreatitis?
A. Pancreatic pseudocyst formation
B. Gallstones
C. Head of pancreas tumour
D. Stricture of bile duct
E. Cholangiocarcinoma
28. Laquelle des propositions suivantes est la cause la plus frquente dictere par cholstase chez
les patients atteints de pancratite chronique?
A. Formation de pseudocyste pancratique
B. Calcul vsiculaires
C. Tumeur de la tte du pancras
D. Stnose du choldoque
E. Cholangio carcinome
29. Concerning cholangiocarcinoma, which of the following is false?
A. Cholangiocarcinoma is less common than gall bladder cancer
B. Cholangiocarcinoma is more common in females
C. There is no association with gallstones
D. Ulcerative colitis is associated with cholangiocarcinoma
E. The parasite Ascaris lumbricoides has been implicated in cholangiocarcinoma.
28. Concernant le cholangiocarcinome, laquelle des propositions suivantes est fausse?
A. Le cholangiocarcinome est moins frquent que le cancer du vsicule biliaire
B. Le-cholangiocarcinome est plus frquent chez les femmes
C. Il n ya pas dassociation avec les calculs vsiculaires
D. La colite ulcrative est associe au cholangiocarcinome
E. Le parasite Ascaris lombricodes a t impliqu dans la gense de chlangiocarcinome
30. A 30 year old cyclist wearing no helmet was hit by a car travelling at 20mph; he was taken
to the local A/E department. On examination he was confused, localizing to pain, and eye,
opening to speech. Which of the following is correct?
A. His Glasgow Coma Score is 11
B. His Glasgow Coma Score is 12
C. Has a mild injury by definition
D. He should receive in-line traction of his cervical spine until cleared radiologically and
clinically
E. He should have a skull X-ray to exclude a linear fracture .
29. Un cycliste de 30 ans portant un casque a t cogn par une voiture roulant la vitesse de
20mph (32km/heure); Il a t amen au service local d9urgences. A lexamen il tait confus,
ragissant la douleur et ouvrant les yeux la parole. Laquelle des propositions suivantes est
exacte?
A.
B.
C.
D.

Son Score de Glasgow est de 11


Son Score de Glasgow est de 12
Il a une lsion minime (lgre) par dfinition
Il doit recevoir une traction de sa colonne cervicale jusqua ce quil devienne
radiologiquement et cliniquement normal.
E. Il doit faire une radiographie du crane pour liminer une fracture linaire

30. A 48 year old diabetic patient has been admitted with diabetic ketoacidosis (DKA). On
examination there is an erythematous boggy area on the left foot over a small ulcer which is the
likely cause of the DKA. What is the management after resuscitating the patient?
A. Ultrasound scan to look for collection
B. CT scan to assess the bones and look for collection
C. Ask for surgical consultation with a view to surgery
D. Magnetic resonance angiogram to assess the soft tissues
E. Plain foot X-ray to look for osteomyelitis
31. Un patient diabtique de 30 ans a t admis avec acidociose diabtique. A lexamen, il existe
une masse fluctuante rythmateuse sur un petit ulcre au niveau du pied gauche
probablement la cause de son acidociose diabtique. Quelle est la prise, en charge aprs
ranimation du patient?
A. Echographie la recherche d'une collection
B. Un scanner pour valuer les os et rechercher une collection
C. Demander une consultation chirurgicale en vue d'une intervention chirurgicale.
D. Angiographie par rsonance magntique pour valuer les tissus mous
E. Radiographie du pied pour rechercher une ostomylite
32. Venous thromboembolism prophylaxis is not required in:
A. A 24-year-old woman undergoing appendicectomy who uses oral contraception
B. A 50-year-old woman with rectal cancer requiring anterior resection
C. An 80-year old man requiring hip replacement
D. A 50-year old woman undergoing palliative bypass for inoperable pancreatic cancer
E. A 40-year old man undergoing inguinal hernia repair
31.La prophylaxie de la maladie veineuse thromlo-embolique n 'est pas requise chez
A.
B.
C.
D.
E.

Une femme de 24 ans sous contraceptif oral subissant une appendicectomie


Une femme de 50 ans prsentant un cancer du rectum e: ncessitant une rsection antrieure
Une femme de 80 ans ayant besoin dune prothse de la hanche
Une femme de 50 ans subissant un by-pass palliatifpour un cancer pancratique inoprable
Un homme de 40 ans subissant une rparation de lhernie inguinale

33. Varicose veins :


A. Are a common cause of pulmonary embolus
B. Usually are symptomatic
C. Are best dealt with by surgery
D. Are generally secondary to deep venous insufficiency
E. Affect 2% of the population
32. Les veines variqueuses
A.
B.
C.
D.
E.

Sont une cause frquente dembolie pulmonaire


Sont habituellement asymptomatiques
Sont mieux traites par la chirurgie
Sont gnralement secondaires une insuffisance veineuse profonde
Affectent 2% de la population

33. Which one of the following is not a recognized long-term complication of partial/total
gastrectomy?
A. Gastric malignancy
B. Obstruction
C. Folate deficiency
D. Iron deficiency
E. Vitamin B12 deficiency
34. Laquelle des propositions suivantes n est pas reconnue comme une complication long terme
dune gastrectomie totale ou partielle?
A. Cancer gastrique
B. Obstruction
C. Dficience en Folate
D. Dficience en Fer
E. Dficience en Vitamine Bu
35. The anatomical site of the neck of a femoral hernia is the
A. Transversalis fascia

B. Ileopectineal ligament
C. Femoral ring
D. Cribriform fascia criteria
E. Obturator foramen
36. Le site anatomique du cou dune hernie fmorale est :
A. Fascia transversalis
B. Ligament pectinal
C. Anneau fmoral
D. Fascia cribriformis
E. Foramen Obturateur
37. A 45 year old patient presents in shock complaining of sudden-onset generalized upper
abdominal pain radiating to the right iliac fossa and the tip of his right shoulder. He reports
one episode of vomiting but none since. He has no past medical problems. On examination his
abdomen is rigid and bowel sounds are absent The diagnosis is:
A. Caecal volvulus
B. Pancreatitis
C. Perforated duodenal ulcer
D. Ascending cholangitis
E. Appendicitis
35. Un homme de 45 ans se prsente en choc en se plaignant dune douleur dinstallation brutale
gnralise de labdomen suprieur et irradiant dans la fosse iliaque droite et la pointe de
lpaule droite. Il na pas dantcdents mdicaux. A lexamen, labdomen est rigide et silencieux
lauscultation. Le diagnostic est :
A.
B.
C.
D.
E.

Volvulus caecal
Pancratite
Ulcre duodnal perfor
Cholangite ascendante
Appendicite
-

36. A 45 year old man presents to the emergency department with a history of coffee-ground
vomiting. He also reports that for 2 days his stool has appeared darker than usual. Which of the
following gives the most sensitive guide to the severity of his gastrointestinal haemorrhage?
A. Haemoglobin
B. Systolic blood pressure
C. Pulse rate
D. Volume of vomitus/melaena
E. Lying and standing blood pressure
36.Un homme de 4S ans se prsente aux urgences avec une histoire de vomissements couleur caf.
Il rapporte aussi que depuis 2 jours ses selles sont apparues plus noires que dhabitude. Laquelle
parmi les propositions suivantes donne Vindication la plus sensible de la svrit de son hmorragie
gastro-inestinalel
A.
B.
C.
D.
E.

Hmoglobine
La pression artrielle systolique
L frquence du pouls
Le volume du vomissement/mlaena
La pression artrielle en position couche et dbout

38. Aspiration pneumonia


A. Most commonly affects the left upper lobe
B. Is more frequent following emergency than elective surgeiy
C. Is less likely if cricoid pressure is used during intubation
D. Is improved by gastric acidity
E. Is less common in patients with reflux oesophagitis
37.La pneumonie daspiration.
A. Plus frquemment affecte le lobe suprieur du poumon guehe
B. Est plus frquente aprs une intervention chirurgicale durgence quaprs lintervention
programme.
C. Est moins probable si une pression cricoidienne est applique lors de Vintubation
D. Est amliore par lacidit gastrique
E. Est moins frquente chez les patients ayant une sopkagite de reflux.
39. Postoperative atelectasis is more common in:
A. Smokers
B. Patients undergoing lower abdominal surgery
C. Non obstructed patients
D. The period between the fifth and post-operative days
E. None of the above
38. Atlectasie post-intervention chirurgicale est plus frquente chez
A. Les tabagiques
B. Ls patients subissant une intervention chirurgicale de labdomen infrieur
C. Les patients nayant pas un syndrome obstructif
D. Pendant la priode allant du 5e au 12e jour post-intervention chirurgicale
E. Aucune de ces rponses nest vraie
39.The following statements concern cardiogenic shock:
A. It caii be distinguished from hypovolaemic shock by central venous blood pressure measurements.
B. It can be distinguished from hypovolaemic shock by systemic blood pressure recordings.
C. It responds to a fluid challenge.
D. It does not respond to inotropic support
E. It is not a cause of post-operative oliguria.
39. Les propositions suivantes concernent le choc cardiognique

A. Il peut tre distingu du choc hypovolemique par la prise de mesures de la pression veineuse
centrale
B. Il peut tre distingu du choc hypovolmique par lenregistrement de la pression artrielle
systmique
C. Il rpond ladministration des soluts
D. Il ne, rpond pas un soutien ionotrope
E. Il nest pas une cause doligurie post-ehirurgicale
40. The following features are in keeping with a postoperative opiate overdose: t
A. Hyperventilation
B. Mydriasis
C. Hypertension
D. Tachycardia
E. Hypoxaemia ' v
40.Les manifestations suivantes sont en rapport avec une over-dose post-chirurgicale dopiacs
A.
B.
C.
D.
E.

Hyperventilation
'Mydriase
Hypertension
Tachycardie
Hypoxemie

41. Which of the following is a contra-indication to major elective surgery requiring general
anaesthesia:
A. A myocardial infarction 12 months ago.
B. A preoperative serum potassium of 2.6mmol/litre in a patient on diuretic therapy
C. Previous mitral valve replacement
D. A known asthmatic patient
E. A known diabetic patient
41. Laquelle des propositions suivantes est une contre-indication une intervention chirurgicale
programme (lective) majeure ncessitant une anesthsie gnrale!
A.
B.
C.
D.
E.

Un infarctus du myocarde datant de plus de 12 mois


Une kalimie pre-opratoire de 2.6 mmol/l chez un patient sous traitement diurtique
Prothese antrieure de la valve mitraie
Un malade asthmatique connu
Un malade diabtique connu

42. Barretts oesophagus is:


A. Unrelated to gasto-oesophageal reflux
B. Best diagnosed using contrast radiology
C. Best treated by surgical resection.
D. A premalignant condition
E. Is not associated with stricture formation
42. L sophage de Barret est
A.
B.
C.
D.
E.

Sans relation avec le reflux gastro-oesophagien


Mieux diagnostiqu par radiologie de contraste
Mieux trait par ablation chirurgicale
Une condition pre-maligne
Nest pas associe a une formation de stnose

43. In Pagets disease of the breast


A. There is usually an underlying intraduct palpilloma
B. Itching is an early symptom
C. An underlying breast mass is a common feature
D. The diagnosis is made primarily on mammography
E. Radiotherapy is the primary treatment
43. Dans la maladie de Paget du sein
A. Ilya habituellement unpapillome intra canalaire sous-jacent
B. Le prurit est un signe prcoce
C. Une masse mammaire sous-jacente est une caractristique frquente
D. Le diagnostic se fait principalement par mammographie
E. La radiothrapie est le traitement principal
44. Which of the following is a classical feature of a perforated peptic ulcer:
A. Succussion splash on examination
B. A long antecedent history
C. Shoulder tip pain
D. Rebound tenderness
E. Presence of bowel sounds
44. Laquelle des caractristiques suivantes est un signe classique dune perforation de lulcre pep
tique?
A.
B.
C.
D.
E.

Bruit de clapotis lexamen


Antcdents longs dulcrepeptique
Douleur la pointe de l'paule
Douleur au rebond ou cri delombilique
Prsence des borborygmes

45. Which of the following statements is true concerning gastric carcinoma:


A. It is twice as common in females as in males
B. It is linked to patients with type O blood group
C. It often presents with a major gastro-intestinal haemorrhage
D. It is sited mainly on the greater curvature of the stomach
E. It can metastasize to the cvaries.
45. Laquelle des propositions suivantes concernant le carcinome gastrique est vraie?
A.
B.
C.
D.
E.

Il est deux fois plus frquent chez la femme que chez lhomme
41
Il est associ aux patients ayant le groupe sanguin de type O
Il se prsente souvent avec une hmorragie gastro-intestinale majeure
Il se localise principalement sur la grande courbure de lestomac
Il peut se mtastaser aux ovaires

46. Strangulating small bowel obstruction is less common in:


A. Small bowel volvulus
B. High small bowel obstruction
C. An incarcerated external hernia
D. Obstructed patients with constant not colicky abdominal pain
E. A closed-loop obstruction
46. Locclusion de lintestin grle par strangulation est moins frquente dans
A. Volvulus de lintestin grle
B. Occlusion haute de, lintestin grle

C. Une hernie incarcre


D. Chez les patients prsentant une occlusion sans douleur constante type de colique
E. Une occlusion boucle ferm
47. In small bowel mesenteric ischaemia:
A. Af absence of intestinal gas is a common finding
B. A history of atrial fibrillation is uncommon
C. The coeliac axis is usually occluded
D. Venous infarction is more common than arterial occlusion
E. None of the above
47.Dans lischmie msentrique de lintestin grle,
A.
B.
C.
D.
E.

Une absence du gaz intestinal est un signe frquent


L'histoire dune fibrillation auriculaire est frquente
Laxe cliaque est souvent bouch ou obstru
Linfarcissement veineux est plus frquent que dans locclusion artrielle
Aucune de ces propositions nest vraie

48. Direct inguinal hernias:


A. Are frequently unilateral 3.
B. Are usually congenital
C. Generally occur in a younger age group than indirect inguinal hernias
D. Often extends into the scrotum '
E. Have a neck that lies medial to the inferior epigastric vessels.
48. Les hernies inguinales directes
A. Sont frquemment unilatrales
B. Sont habituellement congnitales
C. Surviennent gnralement dans une tranche dge plus jeune que dans le cas des hernies
inguinales indirectes.
D. Stendent souvent dans le scrotum
E. Ont leur cou qui se trouve mdialement par rapport aux vaisseaux pigastriques infrieurs.
49. A sliding type inguinal hernia:
A. Can usually be diagnosed postoperatively
B. Will rarely recur
C. Can never contain bladder
D. If unrecognized can lead to bowel injury
E. None of the above
49. Une hernie inguinale par glissement
A. Peut etre diagnostique habituellement en post-opratoire
B. Donnera des rcidives rarement
C. Ne peut jamais contenir la vessie
D. Peut entraner une lsion intestinale si non-diagnostique
E. Aucune de ces propositions n est vraie
50. Which of the following features is characteristic of venous ulcers?
A. They are never painful
B. They are usually found on the medial aspect of the lower leg
C. They are best treated with low compression support hose
D. They usually require hospitalization
E. They are 10 times less common than arterial ulcers.

50. Lequel des aspects suivants est caractristique des ulcres veineux?
A. Ils ne sont jamais douloureux
B. Ils sont habituellement trouvs sur la face interne de la jambe
C. Ils sont mieux traits avec des bas basse compression
D. Ils ncessitent habituellement une hospitalisation
E. Ils sont 10 fois moins frquents que les ulcres artriels
Instructions: Questions 51-56: Choose the correct answer from the following statements / Choisir
la bonne rponse parmi les propositions suivantes.
51. During a cephalic delivery, the normal sequence of the evolution of delivery is
A. Engagement, descent, internal rotation, extension, external rotation.
B. Descent, engagement, extension, external rotation, internal rotation
C. Engagement, descent, internal rotation, external rotation, extension
D. Descent, engagement, extension and external rotation
E. Engagement, external rotation, internal rotation, extension.
51. Pendant un accouchement cphalique, la squence normale de dvolution de laccouchement
est
A.
B.
C.
D.
E.

Engagement, descente, rotation interne, extension, rotation externe


Descente, engagement, extension, rotation externe, rotation interne
Engagement, descente, rotation interne, rotation externe, extension
Descente, engagement, extension et rotation externe
Engagement, rotation externe, rotation interne, extension

52. The exact sequence of the development of the embryo is as follows:


A. Implantation, fertilization, blastocyst formation, ovulation
B. Fertilization, ovulation blastocyst formation, implantation
C. Formation of blastocyst, fertilization implantation, ovulation
D. Ovulation, fertilization, blastocyst formation, implantation
E. Fertilization, blastocyst formation, implantation, ovulation
52. La squence exacte du dveloppement de lembryon est la suivante
A. Implantation, fertilisation, formation du blastocyste, ovulation
B. Fertilisation, ovulation, formation du blastocyste, implantation
C. Formation du blastocyste, fertilisation, implantation, ovidation
D. Ovulation, fertilisation, formation du blastocyste, implantation
E. Fertilisation, formation du blastocyste, implantation, ovulation
53. The commonest method of confirming premature membranes rupture in our hospitals is b
A. Sterile speculum examination alone
B. Sterile speculum examination and Nitrazine Paper Test
C. Ultrasound estimation of amniotic fluid volume
D. Estimation of the glucose and Fructose levels of vaginal secretions
E. Fern test
53. La mthode la plus frquente pour confirmer la rupture prmature des membranes dans nos
hpitaux est par :
A.
B.
C.
D.
E.

Examen au speculum strile seul


Examen au speculum strile et le Nitrazine Paper Test
Estimation par chographie du volume du liquide amniotique
Estimation des taux de glucose et dejrutcse dans les scrtions vaginales,
Fern Test ou test dovulation Claire bleue

54. The complication of -mimetic drugs include one of these symptoms


A. Bilateral pedal oedema
B. Hypertension
C. Palpitations
D. Reduced bi-temporal Visual field
E. Bundle branch bloc
54. La complication des fi-mimtiques comprend un de ces symptmes. Lequel?
A. Oedeme bilatral des pieds
B. Hypertension
C. Palpitations
D. Diminution du champ visuel bitemporal
E. Bloc de branche
55. Approximately what percentage of spontaneous first trimester abortions show chromosomal
abnormalities
A. 1%
B. 10%
C. 30%
D. 60%
E. 80%
55.Dans quel pourcentage approximativement, des avortements spontans rvlent des anomalies
chromosomiques?
A. 1%
B. 10%
C. 30%
D. 60%
E. 80%
56. The commonest organisms causing urinary tract infection in pregnant women is:
A. Staphylococci
B. Streptococci
C. Haemophilis vaginalis
D. Gram negative enteroi jacterium
E. Mycobacteria
56. Parmi les organismes suivants, lequel est le plus frquemment responsable des infections
urinaires chez la femme enceinte?
J,
K,
L,
M,
N,

Staphyloccoques
Streptoccoques
Hemophile Vaginal
Enterobacteries Gram-negatif
Mycobacterie

Instructions: Questions 57-77 Choose the best answer from the different propositions provided
for each question. / Choisissez la meilleure rponse partir des diffrentes propositions donnes
pour chaque question.
57. A 59 year old-postmenopausal woman presents with a months history of per vaginal
bleeding. On speculum examination, the cervix appears normal, and there is no vaginal
discharge. What is the single most appropriate diagnostic test?
A. Cervical smear
B. Hysteroscopy

C. Pelvic ultrasound
D. Serum FSH level
E. Transvaginal ultrasound with endometrial sampling
57. Une femme post-mnopause de 59 ans se prsente avec une histoire de saignement per vaginal
depuis 01 mois. A lexamen au speculum, le col utrin apparat normal et il nya pas dcoulement
vaginal. Quel est le meilleur test diagnostique?
A.
B.
C.
D.
E.

Frottis cervical
Hystroscopie
Echographie pelvienne
Taux de srique de FSH
Echographie transvaginale avec prlvement endometrial

58.A 24 year old woman reports she has missed the first pill in her second pill pack. She had
unprotected sexual intercourse (UPSI) 2 days ago. What is the single most appropriate
management?
A. Advise her to continue as usual as she had UPSI during her pill-free break
B. Advise her to miss the next pill-free break before commencing pack 3.
C. Advise her to take the first and second pills now and continue as normal
D. Fit now an intrauterine contraceptive device (IUCD)
E. Prescribe levonorgestrel l.5mg stat
58. Une femme de 24 ans rapporte quelle na pas pris la premire pilule de sa deuxime boite de
pilules. Elle a eu des rapports sexuels non protgs il ya deux jours. Quelle est le seul moyen
appropri de prise en charge?
A. La conseiller de continuer comme dhabitude tant donn qu 'elle a eu des rapports sexuels
non protgs pendant lintervalle libre o elle ne devrait pas prendre la pilule
B. La conseiller de sauter lintervalle libre suivant avant de commencer avec la troisime boite
C. La conseiller de prendre la premire et la deuxime pilule maintenant t et de continuer comme
dhabitude
D. Insrer maintenant un strilet
E. Prescrire Levonorgestrel J.5mg en une seule prise
59.A 29 year old woman with a family history of breast cancer in her mother at age 35, would
like to be screened for breast carcinoma. What is the single most appropriate management?
A. Offer mammogram
B. Offer ultrasound '
C. Reassurance as no breast symptoms at present
D. Refer to family history breast clinic *
E. Urgent referral to one-top breast clinic
59. Une femme de 29 ans avec antcdents familiaux de cancer du sein chez sa mre lge de 35
ans voudrait faire un dpistage du cancer du sein. Quelle est le seul moyen appropri de prise en
charge?
A. Offrir la mammographie
B. Offrir lchographie
C. Rassurer car il nya pas de symptmes actuellement
D. Rfrer la clinique pour femmes avec antcdents familiaux de cancer du sein
E. Rfrer en urgence a la clinique pour sein
60.A patient experiences dystocia (difficult and prolonged labour) with a breech baby. Her baby
weighs 3.5kg but does not move his right arm. What is the single most likely diagnosis?
A. Clavicle fracture

B. Fracture of the shaft of humerus


C. Fracture of surgical neck of humerus
D. Pulled elbow
E. Shoulder dislocation
60. Une patiente est en dystocie (travail long et prolong), avec prsentation en sige du bb. Son
bb pse 3.5 kg mais ne soulve pas son bras droit. Quel est le diagnostic le plus probable?
A. Fracture de la clavicule
B. Fracture de lhumrus
C. Fracture du col chirurgical de lhumrs
D. Coude tir
E. Luxation de lpaule
61.A 52 year old married woman presents with bleeding per vaginam. She reached menopause at
age 48. Nothing abnormal is discovered on examination of the cervical os. What is the single next
most appropriate investigation?
A. Endocervical swabs and high vaginal swap
B. Pap smear
C. Refer for colposcopy and iodine staining
D. Refer for transvaginal ultrasound
E. Take blood for FSH levels
61. Une femme marie de 52 ans prsente un saignement vaginal. Elle a eu la mnopause lge
de 48 ans. Rien danormal nest observ lexamen du col utrin. Quelle est la seule
investigation la plus approprie ?
A. Prlvements endocervical et vaginal haut
B. Frotus pour Papanicolau (Pap Smear)
C. Rferer pour colposcopie et coloration par liode
D. Rjrer pour chographie transvaginal
E. Prlever le sang pour dosage de FSH.
62. A 45 year old woman reports a green discharge from her right nipple. She is concerned. On
exam, there are no lumps and no axillary lymph & denopathy. There is redness and
tenderness around the nipple. What is the single most likely diagnosis?
A. Breast abscess
B. Duct ectasia
C. Intraductal papilloma
D. Mastitis
E. Pagets disease
62. Une femme de 45 ans rapporte un coulement verdtre de son mamelon droit. Elle est
proccupe. A lexamen, il nya pas de nodules et pas dadnopathie axillaire. Le mamelon est
rouge et douloureux. Quel est le seul diagnostic le plus probable?
A. Abcs du sein
B. Ectasie ductale
C. Papillon intraductale
D. Mastite
E. La maladie de Paget
63. A 30 year old woman with a pregnancy of 30 weeks is involved in a road traffic accident.
What is the single special precaution to take with this woman during the initial assessment?
A. Administer intravenous fluids only when she is haemodynamically unstable.
B. Administer Rh immunoglobulin therapy
'
C. Avoid pelvic x-rays
'
D. Transport her in the left lateral decubitus position with her right leg raised
E. Transport her in the right lateral decubitus position with her left leg raised.

63.Une femme de 30 ans ayant une grossesse de 30 semaines est implique dans un accident de la
voie publique. Quelle est la seule mesure de prcaution spciale prendre chez cette femme au
cours de lvacuation initiale?
A. Administrer les soluts intraveineux seulement lorsquelle est instable sur le plan
hmodynamique
B. Administrer la thrapie par immuno-globuline Rh
C. Eviter les radiographies pelviennes
D. La transporter dans une position de decubitus latral gauche avec sa jambe droite surleve
E. La transporter dans me position de decubitus latral droit avec sa jambe gauche surleve
64. A 55 year old woman post vaginal hysterectomy only complains of constant urine dribbling.
What is the single most likely diagnosis?
A. Bladder outlet obstruction
B. Detrusor failure
C. Stress incontinence
D. Uterine prolapse
E. Vesico-vaginal fistula
64.Une, femme de 55 ans ayant eu une hystrectomie per vaginale ne se plaint que de fuite d urines.
Quel est le seul diagnostic le plus probable?
A.
B.
C.
D.
E.

Obstruction de l'orifice de la vessie


Insuffisance du dtrusor
Incontinence due au stress
Prolapsus utrin
Fistule vesico-vaginal

65. A 34 year old woman with a pregnancy of 11 weeks would like to be screened for Downs
syndrome baby. She is adamant she wants testing now and does not want to wait What is the
single most appropriate screening test?
A. Amniocentesis
B. Chorionic villus sampling
C. Nuchal translucency test
D. Percutaneous umbilical cord sampling
E. Triple blood test; '
65. Une femme de 30 ans avec une grossesse de'11 semaines aimerait faire le dpistage de
syndrome de Down pour sort bb. Elle insiste pour avoir ce dpistage fait maintenant et ne veut
pas attendre. Quel est le seul test de dpistage le plus appropri?
A.
B.
C.
D.
E.

Amniocentse
chantillonnage de villus chorioniques
Test de translucidit nuchale
chantillonnage percutan du cordon ombilical
Test sanguin.triple (Triple blood test)

66. A 15 year old school girl complains of dysrnenorhea. She has a regular cycle. What is the
single most appropriate management?
A. Arrange transvagiiiai ultrasound
B. Prescribe mefanami acid
C. Prescribe noreihisterone
D. Prescribe tranexamic scid
E. Suggest commencing combined oral contraception

66. Une colire de 15 ans se plaint de dysmnorrhe. Elle a un cycle rgulier. Quelle est la seule
prise en charge la plus approprie
A. Prvoir une chograpide transvaginale
B. Prescrire iacide mefammique
C. Prescrire la norethisterone
D. Prescrire lacide transxqmiqus
E. Suggrer quelle commence une contraception combine orale
67. A 40 year old woman with a child with cystic fibrosis would like antenatal testing to ensure
her second child is not affected. What is the single next most appropriate investigation?
A. Amniocentesis ai 5 weeks of pregnancy
B. Chorionic viilus sampling at 19 weeks of pregnancy
C. Couplesgenetic screening
D. Reimplantaiion diagnosis
E. Ultrasound
67.Une femme de: 40 ans ayant un enfant avec mucoviscidose aimerait avoir un dpistage
antnatal pour sassurer que son deuxime enfant ne sera pas affect. Quelle est la seule
investigation l plus approprie?
A.
B.
C.
D.
E.

Amniocentse 15 semaines de grossesse


Echantillonnage de villas chroniques 19 semaines de grossesse
Dpistage gntique du couple
Diagnostic de rimplantation
Echographie.

68. A 40 year old woman has an antenatal serum triple marker test. The alpha foeto-protein and
oestriol levels are low but the hCG is twice normal. What is the single most likely potential
diagnosis?
A. Downs syndrome
B. Hydafiform mole
C. Neural tube defect
D. Normal pregnancy
E. Trisomy 13
68.Une femme de 40 ans a un test triple rique marqueur prnatal. Les taux dalpha-foeto- protine
et doestriol sont bas mais le taux de hCG est de deux fois la normale. Quel est le seul diagnostic le
plus probable?
A.
B.
C.
D.
E.

Syndrome de Down
Mole hydatiforme
Dfaut du tube neural
Grossesse normale
Trisomie 18

69. A 30 year old womans smear test comes back positive for severe dyskaryosis. What is the
single most appropriate management?
A. Refer for colposcopy and biopsy
B. Refer for cone biopsy
C. Refer for LLETZ
D. Repeat smear test in 6 months .
E. Repeat smear test in 1 year
69.Le rsultat du frottis dune femme de 30 ans revient positif pour la dyskaryose svre. Quelle est
la seule prise en charge la plus approprie?

A.
B.
C.
D.
E.

Rfrer pour colposcopie et biopsie


Rfrer pour biopsie conale
\
Rfrer pour LLETZ
Reprendre le frottis dans 6 mois
Reprendre le frottis dans 1 an

70. A 40 year old womans smear test comes back positive for CIN II. .What is the single most
appropriate management?
A. Refer for colposcopy and biopsy
B. Refer for cone biopsy
C. Refer for LLETZ
D. Repeat smear test in 6 months
E. Repeat smear test in 1 year
70.Le frottis dune femme de 40 ans revient positif pour un CINII. Quelle est la seule prise en
charge la plus approprie?
A.
B.
C.
D.
E.

Rfrer pour colposcoph et biopsie


Rfrer pour biopsie conale
Rfrer pour LLETZ
Reprendre le frottis dans 6 mois
Reprendre le frottis dans 1 an

71. A 46 year old woman informs you that her mother was diagnosed with ovarian cancer at age
55. She is concerned as she now smokes, drinks and is HRT (hormono replacement therapy).
What is the single most appropriate management?
A. Advise her to stop drinking as it is a risk factor for ovarian cancer
B. Advise her to stop HRT (hormone replacement therapy) as it doubles the risk of ovarian
cancer
C. Advise her to stop smoking as it is a risk factor for ovarian cancer
D. Refer her for genetic testing.
E. Test for CA-125
71.Une femme de 46 ans vous informe que sa mre a t diagnostiqus avec un cancer de lovaire
lge de 55 ans. Elle est proccupe maintenant car elle fume, consomme lalcool et est sous
thrapie hormonale de remplacement. Quelle est la seule prise en charge la plus approprie?
A. La conseiller darrter de consommer l'alcool car cest un facteur de risque pour le cancer de
lovaire
B. La conseiller darrter la thrapie hormonale de remplacement car elle double le risque de
cancer de lovaire
C. La conseiller darrter de fumer car le tabac est un facteur de risque de cancer de lovaire
D. D. Rfrer pour le dpistage gntique
E. Tester pour CA-125
72. A 45 year old woman reports nipple changes. On examination, there are scaly, erythematous
plaques on and around the inverted nipple. On squeezing the nipple, a serosanguinolante
discharge appears. What is the single most likely diagnosis?
A. Amyloidosis
B. Contact dermatitis .
C. Exzema
D. Mastitis
E. Pagets disease

72. Une femme, 45ans rapporte des modifications sur son mamelon. A lexamen, il ya des plaques
squameuses rythmateuses sur et autour du mamelon invers. En pressant le mamelon, un coulement
srosanguineux apparait. Quel est le seul diagnostic le plus probable?
A.
B.
C.
D.
E.

myloidose
Dermatite de contact
Eczema
Mastile
La maladie de Paget

73. A.25 year old woman would like to know the mode of action of the mini-pill. What is the
single best answer?
A. Alteration of cervical mucus
B. inhibition of implantation of an egg and thickening of cervical mucus
C. Inhibition of ovulation and implantation.
D. Inhibition of ovulation
E. Spermicidal
73.Une femme de 25 ans aimerait connatre le mcanisme daction de la minipilule. Quel est la
seule meilleure rponse ?
A.
B.
C.
D.
E.

Altration ou modification du muscul cervical


Inhibition de limplantation d l uf et paississement du mucus cervical
Inhibition de lovulation et d'implantation
Inhibition de lovulation
Spermicide

74. A 40 year old woman with a 34 weeks pregnancy is noted to have an incidental BP of
160/100mmHg on an antenatal review. She denies blurring vision. There is no proteinuria and
no signs of peripheral oedema. Blood tests are normal. What is the single most appropriate
management?
A. Admit to hospital
B. Commence hydralazine
C. Commence labetalol ,
D. Commence methyldopa
E. Commence nifedipine
74. Une femme de 40 ans avec une grossesse de 34 semaines a une tension artrielle de
160/l00mmHg de dcouverte fortuite lors de la visite prnatale. Elle rfute davoir la vision
floue. Il ny a pas de protinurie et il nya pas de signes ddme priphrique. Les tests
sanguins sont normaux. Quelle est la seule prise en charge la plus approprie?
A. Faire admettre lhpital
B. Dbuter le traitement avec hydralazine
C. Dbuter le traitement avec Labetalol
D. Dbuter le traitement avec methyl dopa
E. Dbuter le traitement avec nfidepine
75. A 40 year old woman is brought in emergency with, shock and profuse vaginal bleeding. She
had delivered her third baby at home. On examination a low transverse scar is noted. What is
the single most likely diagnosis?
A. Fibroid Uterus
B. Postpartum haemorrhage
C. Rhesus incompatibility
D. Ruptured ectopic pregnancy

E. Uterine rupture
75. Une femme de 40 ans est amene aux urgences dans un tat de choc et saignement profits per
vaginal. Elle a accouch son 3e bb la maison. A lexamen, une cicatrice transversale basse est
observe. Quel est le seul diagnostic le plus probable ?
A.
B.
C.
D.
E.

Utrus fibromateux
Hemorragie du post partum
Incompatibilit rhsus
Grossesse extra-utrine
Rupture de lutrus

76. A 6 weeks pregnant woman presents with right-sided pelvic pain. She has not had a scan for
this pregnancy. There is no per vaginal bleed. She presents to casualty at 2 an with a BP of
100/60mmHg and a pulse of 80/minute. What is the single most appropriate management?
A. Give her a form for an outpatient ultrasound
B. Refer her to the gynaecologist on call
C. Keep her in casualty for observation
D. Take high vaginal and endo-cervical swabs.
E. Tell her to return at 9a.m to the early pregnancy unit for a scan
76. Une femme enceinte de 6 semaines se prsente avec des douleurs pelviennes droites. Elle na pas
encore eu faire une chographie pour cette grossesse. Il ny a pas de saignement vaginal. Elle se
prsente aux urgences 2 heures du matin avec une tension artrielle de l00/60mmHg et un pouls
80/minute. Laquelle est la seule prise en charge la plus approprie?
A.
B.
C.
D.
E.

Lui donner un bulletin d'examen dchographie faire en externe


La rfrer un gyncologue dz garde
La garder lurgence pour observation.
Prendre des prlvements endocervical et vaginal haut
Lui (lire de revenir 9 heures le matin la .consultation pour les femmes sen dbut de
grossesse pour une chographie,

77. A 30 year old woman complains of 7 months of amenorrhea, facial hair and acne. Site has
put on 2kg in the past month. What is the single most likely diagnosis?
A. Adrenal tumour
B. Cushings disease
C. Idiopathic
.
D. Ovarian carcinoma
E. Polycystic ovarian disease
77. Une femme de 30 ans se plaint dune amnorrhe de 7 mois, hirsutisme et acn. Elle a pris 2kg
de poids au cours du dernier mois. Quel est le seul diagnostic le plus probable?
A. fumeur adrnale
B. La maladie de Cushing
C. ldiopathique
D. Carcinome ovarien
E. Maladie polykystique de lovaire
Instructions; Questions 78-96 :Choose the single most appropriate diagnosis from the list of
options proposed. / Choisir parmi les propositions suivantes la reponse approprie.
78.A 31 year old woman with complete placenta preavia was delivered by caesarian section. Two
hours later, she is noted to have significant post partum haemorrhage
A. Placenta accreta . .

B. Implantation in the lower, uterine segment


C. Uterine rapture
D. Thrombocytopenia
E. Ruptured vsa praevia
78. Une femme de 31 ans avec un placenta prvia complta t accouche par csarienne. Deux
heures plus tard, il est observ qu elle a une hmorragie importante du post partum. Quel est
le seul diagnostic le plus probable?
A. Placenta accreta
B. Implantation dans le segment utrin infrieur
C. Rupture de lutrus
D. Thrombocytopnie
E. Rupture de vasa prraevia
79. Following a spontaneous vaginal delivery, a 22 year old woman continues to bleed in spite of
the use of oxytocin. The uterus appears to contract well but then relaxes with increase
bleeding.
A. Retention of a succenturiate lobe
B. Cervical laceration
C. Placenta secreta
D. Atonic placenta
E. Thrombocytopenia
79. Suite un accouchement spontan par voie vaginale, une femme de 22 ans continue de
saigner malgr lutilisation de locytocine, Lutrus parait bien se contracter mais se relche
avec une augmentation de saignement. Quel est l diagnostic le plus probable?
A. Rtention dun lobe succenturi
B. Dchirure cervicale
C. Placenta accreta
D. Atonie placentaire
E. Thrombocytopnie
80. A 27 year old woman whose last period was 8 weeks ago presents with heavy vaginal
bleeding and left lower quadrant pain. Serum P-HCG is slightly elevated. Vaginal
examination shows a closed cervical os.
A. Hydatiform mole
B. Threatened miscarriage
C. Ectopic pregnancy
D. Bleeding corpus luteum
E. Androgen excess amenorrhoea
80. Une femme de 27 ans qui avait eu-ses dernires rgles il ya 8 semaines, prsente un saignement
vaginal abondant et des douleurs dam lhypogastre gauche. Le taux srique de p- HCG est
lgrement lev. Lexamen vaginal rvle un col utrin ferm. Quel est le seul diagnostic le plus
probable?
A. Mole hydatiforme
B. Menace davortement
C. Grossesse extra-utrine
D. Saignement du corps jaune
E. Amnorrhe par excs dandrognes
81. A 14 years old adolescent with normal sexual development complains of amenorrhoea for 5
months. Her first menses were 10 months ago, since which she has had three menses.
A. Androgen excess amenorrhoea
B. Hypogonadotrophic amenorrhoea
C. Physiological amenorrhoea

D. Eugonadotrophic amenorrhoea
E. Hypergonadotrophic ameorrhee
81.Une adolescente de 14 ans avec un dveloppement sexuel normal se plaint dune amnorrhe
depuis 5 mois. Ses premires rgles ont eu lieu il ya 10 mois. Depuis lors elle a eu 3 menstrues.
Quel est le seul diagnostic le plus probable?
A.
B.
C.
D.
E.

Amnorrhe par exces dandrogenes


Amnorrhe hypogonadotrophique
Amnorrhe physiologique
Amnorrhe eugonadotrophique
Amnorrhe hypergonadotrophique

82. A 28 year old woman on combined oral contraceptive pills presents with a 3 month history
of post coital and intermenstrual bleeding. Speculum examination shows an everted
ulcerated cervix. An endoeervicai smear is normal.
A. Foreign body
B. Endometriosis
C. Cervical polyp
D. Cervical ectropion
E. Cervical carcinoma
82.Une femme de 28 ans sous pilule contraceptive combine orale prsente une histoire dun
saignement post-cutal et intermenstruel voluant depuis 3 mots. Lexamen par spculum rvle un
col ulcr et en version. Le frottis endocervical est normal. Quel est le seul diagnostic le plus
probable?
A.
B.
C.
D.
E.

Corps tranger
Endomtriose
Polype cervical
Ectropion cervical
Carcinome cervical

83. A 59 year old obese woman presents with post-menopausal bleeding. She had regular
periods until 3 years ago. General and speculum examination reveals no abnormality.
Vaginal ultrasound shows endometrial thickening.
A. Endometriosis
B. Cervical carcinoma
C. Endometrial carcinoma
D. Bleeding disorder
E. None of the above
83. Une femme obse de 59 ans se prsente avec un saignement post mnopause. Elle avait des
rgles rgulires jusqua il ya 3 mois. Lexamen gnral et au spculum ne rvle aucune
anomalie. Une chographie vaginale rvle un paississement de lendomtre. Quel est le seul
diagnostic le plus probable?
Endomtriose
Carcinome du col utrin
Carcinome de l'endomtre
Trouble de saignement
Aucune des rponses ci-dessus
84. A 27 year od woman with a past history of pelvic inflammatory disease and amenorrhoea for
3 weeks presents with left iliac fossa pain. She reports a small amount of brown watery vaginal
discharge.
A.
B.
C.
D.
E.

A. Ectopic pregnancy
B. Threatened miscarriage
C. Missed abortion
D. Septic abortion
E. Pelvic inflammatory disease
84. Une femme de 27 ans avec des antcdentsdune infection pelvienne et une amnorrhe de 8
semaines prsente une douleur de la fosse iliaque gauche. Elle rapporte aussi un coulement
vaginal liquidien et bruntre de petite abondance. Quel est le seul diagnostic le plus probable?
A. Grossesse extra utrine '
B. Menace avortement
C. Grossesse arrte
D. Avortement septique
E. Infection pelvienne (pelvic inflammatory disease)
85. A 17 year, od high school student presents 1 week before her exams having missed her
period in addition to recurrent colicky abdominal pain and diarrhea. General examination
is normal as are blood tests. Pregnancy test is negative.
F. Ectopic pregnancy
G. Ovarian cyst torsion
H. Endometriosis
I. irritable bowel syndic me
J. Appendicitis
85.Une lycenne de 17 ans se prsente une semaine avant ses examens nayant pas eu ses rgles en
plus de douleurs abdominales rcurrentes type de colique et la diarrhe. Lexamen gnral est
normal ainsi que les tests sanguins. Le test de grossesse est ngatif. Quel est le seul diagnostic le plu
; probable ?
A.
B.
C.
D.
E.

Grossesse extra utrine


Torsion dan kyste ovarien
Endomtriose
Syndrome dintestin irritable ou colopathie fonctionnelle
Appendicite

86. A 34 year old woman presents with a 2 year history of gradually worsening painless
menorrhagia. On examination, she is pate and has an enlarged bulky uterus.
A. Endometrial carcinoma
B. Bleeding disorder
C. Clotting disorder
D. Uterine fibroids
E. Adenomyosis
86. Une femme de 54 ans se prsente avec une histoire de 02 ans de mnorragie non douloureuse
qui saggrave progressivement, A lexamen, die prsente une pleur et un utrus volumineux. Quel
est le seul diagnostic le plus probable!
A. Carcinome de lendomtre
B. Trouble de saignement
C. Trouble de coagulation.
D. Fibrome utrin
E. Adnomyose
87. A 32 year old woman who is 19 weeks pregnant presents with severe nausea, vomiting and
heavy vaginal bleeding. On examination the uterus is large for date.

A. Ectopic pregnancy
B. Hydatiform mole
C. Choriocarcinoma
D. Abruptio placenta
E. Placenta praevia
87. Une femme de 32 ans qui est enceinte de 19 semaines se prsente avec des nauses importantes,
vomissements et saignement vaginal abondant. A lexamen, lutrus est plus volumineux pour la
dure de la grossesse. Quel est le seul diagnostic le plus probable?
A. Grossesse extra utrine
B. Mole hydatiforme
C. Choriocarcinome
D. Abruptio placentae
E. Placenta praevia
88. A 23 year old woman presents with pelvic pain, fever and vaginal discharge. On examination
there is marked lower and adnexal tenderness. There is also cervical motion tenderness.
A. Ectopic pregnancy
B. Septic abortion
C. Pelvic inflammatory disease
D. Degenerating fibroid
E. Endometriosis
88.Une femme de 23 ans prsente une douleur, pelvienne, fivre et coulement vaginal. A lexamen,
il ya une douleur intense la palpation du bas ventre et des annexes, La mobilisation du col utrin
est aussi douloureuse. Quel est le seul diagnostic le plus probable.
A.
B.
C.
D.
E.

Grossesse extra utrine


Avortement septique
Infection pelvienne (Pelvic inflammatory disease)
Dgnrescence des fibromes
Endomtriose

89. A 37 year old para 3+1 who is 34 weeks pregnant presents with painless fresh vaginal bleeding
of about 150cc in volume.
A. Placenta praevia
B. Abuptio placenta
C. Bleeding disorder
D. Clotting-disorder
E. Cervical erosion
89. Une femme de 37 ans, para 3+1 qui est enceinte de 34 semaines prsente un saignement vaginal
frais non douloureux dun volume denviron ISO cc. Quel est le seul diagnostic le plus probable?
A.
B.
C.
D.
E.

Placenta praevia
Abruptio'placenta
Trouble de saignement
Trouble de coagulation
Erosion cervicale

90. A 37 year old woman presents with secondary amenorrhoea, palpitations and weight loss
of5kg despite a good appetite. Pregnancy test is negative.
A. Primary ovarian failure
B. Hypothyroidism
C. Thyrotoxicosis

D. prolactinoma
E. Polycystic ovary syndrome
90. Une femme de 37 ans prsente une amnorrhe secondaire, palpitations et perte de poids de
5kg malgr un bon apptit. Le test de grossesse est ngatif. Quel est le seul diagnostic le plus
probable?
A. Insuffisance ovarienne primitive
B. Hypothyrodisme
C. Thyrotoxicose
D. Prolactinome
E. Syndrome ovarien polykystique
Instructions; Q u e s t i o n s : 9 1 - 9 3 : for each question below, choose the single most
appropriate investigation from the list of options proposed. / Pour chaque cas ci-dessous,
slectionnez la seule investigation la plus approprie parmi les propositions suivantes.
91. A 26 year old woman \ presents with amenorrhoea for weeks vaginal spotting and mild right
lower quadrant pain. Her periods have always been irregular. On examination, the uterus is
of normal size and the right lower quadrant is tender. Her -HCG level the day before was
1100mlU/ml.
A. Laparoscopy
B. Measurement of progesterone levels
C. Measurement of gonadotrophin levels
D. Measurement of TSH levels
E. Pelvic ultrasound scan
91.Une femme de 26 ans prsente une amnorrhe de 7 semaines, saignement vaginal et lgre
douleur dans le qmdrant abdominal infrieur droit. Sss rgles ont t toujours irrgulires. A
lexamen, lutrus est de taille ncr.i.ale et le palpation du quadrant abdominal infrieur droit est
douloureuse. Son taux de fS-HCG la veille tait de 1100mIU/ml.
A.
B.
C.
D.
E.

Laparoscopie
Dosage de taux de progestrone
Dosage de taux de gonadotrophine
Dosage de taux de TSH
Echographie pelvienm

92. Investigating a young female with dysuria, vaginal discharge and pelvic pain
A. Vaginal wall smear
B. Cervical mucus studies
C. Temperature chart
D. Endometrial biopsy
E. Chlamydia testing
92. Investigation dune jeune femme prsentant une dysurie, un coulement vaginal et des douleurs
pelviennes.
93. Frottis de la paroi vaginale
94. Etude du mucus cervical
95. Feuille de temprature
96. Biopsie de lendomtrite
97. Tests pour la recherche de chlamydia
93. A 23 year old nulligravida stopped her oral contraceptive pills to conceive. She had a
menstrual flow after the last pack of contraceptive pills, and then was in amenorrhoea for 7
months. She is otherwise fit and healthy.

A. Measurement of serum prolactin levels


B. Measurement of gonatrophin levels
C. Measurement of TSH levels
D. Measurement of progesterone levels
E. Measurement of -HCG level.
93. Une nulligravide de 23 ans a arrt ses pilules contraceptives pour concevoir. Elle a eu ses
rgles aprs la dernire boite des pilules et est devenue amnorrhique pendant 7 mois. Elle est
par ailleurs en trs bonne sant.
A. Dosage du taux srique de prolactine
B. Dosage du taux de gonadotrophines
C. Dosage du taux de TSH
D. Dosage du taux de progestrone
E. Dosage du taux de -HCG
Instructions: Questions 94-100 : For each case below, choose the single most appropriate
management step from the list of options proposed. / Pour chaque cas ci-dessous, choisissez
laseule prise en charge la plus approprie parmi les options proposes.
94. A 60 year old woman presents with a hard lump in the lower inner quadrant of the right
breast. On examination axillary lymphadenopathy is palpable. Mammography showed a 2.5
cm mass with microcalcification.
A. Invasive ductal carcinoma
B. Fibroadenoma
C. Breast cyst
D. Intraductal papilloma
E. Fibroadenosis ,
94.Une femme de 60 ans prsente une masse dure dans le quadrant interne et infrieur du sein
gauche. A lexamen, une adnopathie axillaire est palpable. La mammographie rvle une masse de
25 cm avec des microcalcifications.
A.
B.
C.
D.
E.

Carcinome ductal invasif


Fibro adnome,
Kyste du sein
Papillome intraductal
Fibro adenose

95. A 31 year old primaparous woman is fully dilated for 4 hours and has been pushing for
1.5hoiirs. The foetal head is in occipito-posterior position 4cm below the ischial spine.
A. Oxytocin
B. intravenous fluids
C. Emergency caesarian section
D. Oxygen
E. Forceps
95. Une femme primipare de 31 ans u eu une dilatation compltedu col depuis 4 heures. Elle
pousse depuis 1 heure 30 minutes. La tte ftale est duns une position occipito-postrieure fi
4 cm au dessous de lpine sciatique.
A. Oxytocine
B. Soluts intraveineux
C. Csarienne en urgence
D. Oxygne
E. Forceps

96. A 34 year old obese multiparous woman presents in early labour. On examination the
presenting part has not engaged to the pelvic outlet.
A.
B.
C.
D.
E.
96.
A.
B.
C.
D.
E.

Regular observation
Emergency caesarian section
Oxytocine
Ultrasound scan
Intravenous fluids
Une femme obse et multipare de 34 ans se prsente au dbut du travail. lexamen la partie
qui se prsent nest pas engage dans le dtroit infrieur.
Observation rgulire
Csarienne en urgence
Oxytoine
Echographie
Soluts intraveineux

97. A 37 year old woman presents with a 3 m o n t h h i s t o r y of meno-metrorrhagia. She has been
on oral contraceptive pills.
A. Endometrial biopsy
B. Oral iron therapy
C. Laparoscopy
D. Dilatation and curettage
E. Tranexamic acid
97.Une femme de 37 ans se prsente avec une histoire de 3 mois de mno-mtrorragie. Elle a t
SOUS pilule contraceptive orale.
A.
B.
C.
D.
E.

Biopsie de Vendomire
Thrapie par le fer oral
Laparoscopie
Dilatation et curetage
Acide tranexamique

98. A 27 year old primaparous woman is admitted to hospital 1 hour after rupture of her
membranes. She is 39 weeks pregnant but the baby is known to be small for date.
A. GTG monitoring
B. Ultrasound scan
C. Emergency caesarian section,
D. Regular observation
E. Oxytocin
98. Une femme primipare de 27 ans est admise lhpital une heure aprs la rupture de ses
membranes. Elle est enceinte de 39 semaines mais le bb est connu pour tre petit pour la date.
A.
B.
C.
D.
E.

Monitoring par CTG


Echographie :
Csarienne en urgence
Observation rgulire
Oxytocine

99.A 32 year old multiparous woman is fully dilated for 1 hour and the CTG shows variable
decelerations.
A. Oxygen
B. Intravenous fluids

C. Foetal blood sampling


D. Ultrasound scan
E. Regular observation
99. Une femme multipare de 32 ans est compltement dilate depuis 1 heure et le CTG montre des
dclrations variables
A. Oxygne
B. Soluts intraveineux
C. Echantillonnage du sang ftal
D. chographie
E. Observation rgulire
100.
A 23 year old air hostess with a history of migraine consults her General Practioner
asking for contraception for at least 1 year. Her partner refuses to use condoms.
A. Norplant
B. Depo-provera (progesterone depot injection)
C. Combined oral contraceptive pill
D. Spermicide
E. Progesterone only contraceptive pill
100. Une htesse de lair de 23 ans avec une histoire de migraines consulte son mdecin gnraliste
demandant une contraception pour au moins 1 an. Son partenaire refuse lutilisation des
prservatifs.
A.
B.
C.
D.
E.

Norplant
Depo-provera (injection de depot de progestrone)
Pilule contraceptive combine orale
Spermicide
Pilule contraceptive contenant seulement la progestrone
FIN/END

Vous aimerez peut-être aussi